Board Review

Réussis tes devoirs et examens dès maintenant avec Quizwiz!

When prescribing NSAIDs, such as ibuprofen, it is important to remember that these medications have a maximum dose at which increasing the dose does not result in better pain control and may result in increased side effects. This is known as:

The analgesic ceiling effect of a drug refers to the dose beyond which there is no additional analgesic effect. Higher doses do not provide any additional pain relief but may increase the likelihood of side effects as well as the cost of treatment. This concept, often disregarded in the treatment of pain in the emergency department (ED), should be carefully considered when using common analgesics such as acetaminophen, ibuprofen, and opioids.

A 30-year-old female patient G2P0010 presents to the office complaining of vaginal bleeding and cramping. Currently 10 weeks pregnant, she lost her previous pregnancy to a spontaneous miscarriage at 10 weeks. Cervical exam shows moderate bleeding from the cervix that is dilated. What is the most likely diagnosis?

An inevitable abortion has dilation of the cervix without expulsion of the products of conception. A threatened abortion is defined as vaginal bleeding prior to 20 weeks gestation with a closed cervix. An incomplete abortion is defined as the expulsion of some of the products of conception but not all of them. A habitual abortion is defined as three or more spontaneous pregnancy losses in a row.

Which of the following is NOT a symptom associated with an ovarian cyst rupture?

An ovarian cyst rupture is characterized by sudden, unilateral, sharp pelvic pain. Rupture can also cause peritoneal signs, abdominal distention, and vaginal bleeding.

A 75-year-old G4P4004 slightly obese patient presents to your office with a complaint of increasing pressure in her vaginal area. She describes a feeling as though something is falling out of her vagina. She states that when she walks or stands for longer periods of time it seems to worsen the feeling of pressure. On exam, the vaginal and vulvar areas appear normal. Which of the following would you do next to confirm the most likely diagnosis?

The complaint suggests uterine prolapse. The best way to confirm this diagnosis to ask the patient to bear down (Valsalva) to see if there is a protrusion from the introitus. A uterine prolapse would produce a firm, muscular mass in the vagina or protruding from the vagina.

A 38 year old man presents with Chvostek's sign (facial spasm when tapping on the cheek over the facial nerve). He is found to have a prolonged QT interval on EKG. These findings are suggestive of what underlying endocrine disorder?

The Patient has hypocalcemia. Parathyroid hormone secretion is controlled by serum calcium level and vice versa (i.e. not by a pituitary trophic hormone as with most endocrine glands).Thus, inadequate PTH production due to hypoparathyroidism of any cause (post-surgical, autoimmune, etc) leads to hypocalcemia. The other disorders are not associated with hypocalcemia - in fact, primary hyperparathyroidism causes hypercalcemia.

Which is the most frequent site that female breast cancers are found?

45% of breasts cancers are found in the upper outer quadrant. 25% are found under the nipple and areola area.

Which of the following is the MOST effective type of emergency contraception?

A copper IUD is up to 99.9% effective as an emergency contraceptive method. A benefit is that it can remain in place and be used as a long-term, reversible birth control method. The other choices are effective as well.

Which of the following is required to make a diagnosis of Autism Spectrum Disorder?

A persistent deficit in social communication is the correct answer. Sensitivity to light, sound, or other sensations, repeating words, and mental retardation are often seen in kids with Autism Spectrum Disorder (ASD), but are not part of the diagnostic criteria. Genetic testing is often performed, but most children with ASD do not have an identifiable genetic marker. A diagnosis of ASD requires two things: (1) a persistent deficit in social communication and social interactions; AND (2) restricted, repetitive behaviors, interests, or activities.

A 34-year-old woman presents to you with a 3-day history of a rash on her arms and painful sores in her mouth. Upon examination of the skin, erythematous target shaped papules and vesicles are noted over her distal arms. Vesicles and erosions are noted over the palate and gingiva. Which of the following historical information is the most likely cause of her eruption?

A recent history of herpes simplex viral infection is a common cause of erythema multiforme minor. Prophylactic treatment with acyclovir or valacyclovir, may prevent recurrence of erythema multiforme minor. Incorrect answers: Erythema multiforme minor is not considered contagious, therefore exposure to contacts with this condition is not likely to result in disease transmission. Erythema multiforme minor has not been linked to sensitization to allergens such as plants or skin care products, but rather recent exposure to medications and recent infections. Most cases of erythema multiforme minor are idiopathic.

A 26-year-old male fell with his arm externally rotated and abducted. He presented to the ED complaining of pain, inability to move his shoulder, and shoulder deformity. He was diagnosed with an anterior shoulder dislocation and was appropriately reduced. At his follow up visit to the orthopedic clinic, he complains of numbness on the upper lateral aspect of his arm, as well as an inability to abduct his arm. What are you concerned about? A. Axillary nerve injury B Rotator cuff tear C. Humerus fracture D. Biceps tendon injury

A. Axillary nerve injury A. Axillary nerve injury: an axillary nerve injury is a known complication of a shoulder dislocation. The axillary nerve innervates the deltoid and teres minor, and thus an injury to the nerve could cause diminished sensation along the deltoid as well as abduction and external rotation weakness B. Rotator cuff tear: it is possible to have a concomitant rotator cuff injury, but this is much more likely to be seen in the 50+ age group C. Humerus fracture: if the proximal humerus was fractured, imaging from the ED would have shown this injury D. Biceps tendon injury: a biceps tendon injury is more consistent with a sudden contraction of the biceps while flexion is being resisted (ex. catching a couch while moving). This mechanism of injury is inconsistent with a biceps tendon rupture.

Of the following, which is the most appropriate first line therapy for otitis media?

The AAP currently recommends amoxicillin as the first line therapy for otitis media; amox/clav is a reasonable alternative if concerns about resistant gram negative infection are present. Trimethoprim/sulfa and azithromycin both have poor pneumococcal coverage; clindamycin has poor coverage of H flu and Moraxella. Ceftriaxone can be used to treat otitis if oral medications cannot be tolerated or if they fail, but shouldn't ordinarily be the first choice.

Which of the following is NOT true concerning ectopic pregnancies?

An ectopic pregnancy is the implantation of an embryo anywhere else except in the endometrium of the uterus. 95% of ectopic pregnancies occur in the fallopian tubes with the majority of those occurring in the ampulla. Progesterone only contraception is a risk factor for ectopic pregnancies because the progesterone can thicken the mucous in the fallopian tubes and slow the movement of the egg. The classic triad is delayed menses, vaginal bleeding, and pelvis pain. Beta HCG levels will NOT double every 48-72 hours like in a normal viable pregnancy. In an ectopic pregnancy, the levels will rise but at a much slower rate and will not double every 48-72 hours.

A 24 year old female presents to the ED with a cat bite to her left hand this morning while trying to get her pet cat to the vet in a carry crate. The left hand has a clean puncture wound on the dorsum and scratches on the forearm. The wound is irrigated, cleansed and dressed. The patient is uptodate with tetanus prophylaxis and has no allergies. What preventive oral medication should the patient be prescribed?

According to 2014 dog and cat bite treatment guidelines from the Infectious Diseases Society of America, preemptive early antimicrobial therapy for 3-5 days is recommended for the following patients Immunocompromised Asplenic Anvanced liver disease Have preexisting or resulting edema in the affected area Have moderate-to-severe injuries (especially to hand and/or face) Have injuries that may have penetrated the periosteum or joint capsule The goal of initial therapy is to cover staphylococci, streptococci, anaerobes, and Pasteurella species. Prophylactic antibiotics may be given for a 3- to 5-day course. If the wound is infected on presentation, a course of 10 days or longer is recommended. The first-line oral therapy is amoxicillin-clavulanate. For higher-risk infections, a first dose of antibiotic may be given intravenously (ie, ampicillin-sulbactam, ticarcillin-clavulanate, piperacillin-tazobactam, or a carbapenem). Other combinations of oral therapy include cefuroxime plus clindamycin or metronidazole, a fluoroquinolone plus clindamycin or metronidazole, sulfamethoxazole and trimethoprim plus clindamycin or metronidazole, penicillin plus clindamycin or metronidazole, and amoxicillin plus clindamycin or metronidazole; a less effective alternative is azithromycin or doxycycline plus clindamycin or metronidazole.

A 48 year old non- diabetic man presents with recurrent episodes of early morning weakness, sweating and altered mental status. In the ED he is found to be hypoglycemic with hyponatremia, and hyperkalemia His diet is normal. Which of the following conditions should be considered?

Addison's Disease is adrenal insufficiency. Because cortisol is a counter-regulatory hormone which maintains serum glucose in the fasting state, insufficient cortisol causes fasting hypoglycemia. Primary adrenal insufficiency is associated with hyperpigmentation, orthostatic hypotension, and fasting hypoglycemia.

A 68 year old female had an abdominal CT a week ago for abdominal pain that resolved. She presents with an incidentally found right kidney upper pole complex cyst measuring 4.5 cm with internal enhancement and septations. What options should be discussion with patient?

An enhancing, 4.5 cm renal mass with septations is renal cell carcinoma until proven otherwise. It depends on the location of the mass but radical nephrectomy, partial nephrectomy and ablation should all be discussed with the patient as potential treatments. They all carry separate risks and benefits that may be better or worse for your patient.

A 41 year old woman presents with acute abdominal pain. She has a history of multiple previous abdominal surgeries (appendectomy, cholecystectomy, exploratory laparotomy for endometriosis). On physical exam of her abdomen you note high pitched tinkling bowel sounds during auscultation. Which of the following is most likely the etiology of her condition?

Adhesions are bands of scar tissue that form between organs. In the abdomen, they form after an abdominal surgery or after a bout of intra-abdominal infection. More than 95% of patients who undergo abdominal surgery develop adhesions; these are almost inevitably part of the body's healing process. Although most adhesions are asymptomatic, this is the most common cause of bowel obstructions, infertility, and chronic pain. In a study that reviewed over 18,912 patients who underwent previous open abdominal surgery, 14.3% presented with a bowel obstruction in 2 years, with 2.6% of these patients requiring adhesiolysis to relieve the bowel obstruction. Postoperative adhesions account for 74% of cases of small-bowel obstruction. Colonic neoplasms often present with anemia, weight loss, bowel changes, possible family history of colon cancer or polyps. Diverticulitis often presents with lower left quadrant pain, changes in bowel habits (diarrhea, constipation, bleeding). Patients may have fever and an elevated white blood cell count. Inguinal hernia presents more often in males often with a history of lifting or straining, a pouch in the groin, ache in the inguinal region. Pyelonephritis often presents in females with a history of UTI or urethritis or dysuria and/or urinary frequency, hematuria, fever, flank pain.

When prescribing opiate pain medication for several days, you must consider:

All of these must be considered. adverse effects of opioids include central nervous system depression, hypotension, respiratory depression, constipation, nausea, and itching. The US Food and Drug Administration (FDA) recently mandated opioid labeling changes to highlight several of the less-recognized effects, including serotonin syndrome, adrenal gland suppression, and decreased sex hormone levels

A 47 year old woman is found to have an elevated serum TSH. Which of the following clinical findings is she most likely to have?

All the findings listed above are common in HYPERthyroidism but only fatigue is common in HYPOthyroidism as well.

Which of the following statements is false regarding the use of opioids for in hospital pain management?

All the statements are true about opioid use, except dosing it as needed (PRN) or when the patient requests it. The goal in an in hospital pain treatment plan with medication is to obtain a steady blood level of analgesic that is safe. PRN dosing causes a roller coaster effect of analgesia, then pain. Opiates should be administered by a PCA pump or given on the half life of the drug for optimum safe analgesia.

Which muscle is the main contributer to the "rotator cuff"?

All these muscles contribute to shoulder function, but the supraspinatus along with the infraspinatus, subscapularis and teres minor comprise the rotator cuff (SITS).

A 32 year old man involved in a motor vehicle crash is brought to the Emergency Department by emergency medical services. During the trauma evaluation it is noted that he has blood at the urethral meatus. Which of the following studies should be obtained to evaluate for urethral injury?

Although CT and FAST would likely be obtained to assess for other associated injuries neither is used to evaluate the urethra. Retrograde urethrography is the standard imaging study for the diagnosis of urethral injury. It is performed using gentle injection of 20-30 mL of contrast into the urethra while occluding the meatus, with a balloon of a Foley catheter inflated in the fossa navicularis. Films should be taken in a 30°-oblique position, unless this is not possible because of the severity of the pelvic fractures and associated patient discomfort. A urethrogram allows for identification of the site of injury and assessment of the extent of any injury. Any extravasation outside the urethra is pathognomonic for urethral injury.

A 16-year-old girl presents to your office for evaluation and management of her acne. Her mom is asking about that factors that may be contributing to the cause of her acne. Which of the following has NOT been proven to cause acne?

Although dairy consumption has been associated with acne, research has not proven that the consumption of chocolate, sodas and greasy foods is associated. The remaining answers ARE associated with acne: hormones (androgen excess), emotional stress and Propionibacterium acnes bacteria are all common causes of acne.

Which of the following is not considered a long-term complication of Pelvic Inflammatory Disease or PID?

Although you can see a vaginal discharge with PID, it is not a long-term complication. Infertility, ectopic pregnancy, and chronic pelvic pain are all long-term complications from PID usually due to the inflammation and subsequent adhesions and scarring it produces in the pelvic cavity.

A 72 year old woman presents to the emergency department with severe abdominal pain and vomiting that began abruptly one hour ago. She has a history of atrial fibrillation and hypertension but has otherwise been healthy.She has never had abdominal surgery.On physical exam she appears to be extremely uncomfortable but her abdomen is soft, without guarding or rebound tenderness. Which of the following is the most likely diagnosis?

Appendicitis typically presents in a younger patient presenting with vague periumbilical pain that then localizes to the RLQ over several hours. It is accompanied by nausea/vomiting and low grade fever and slightly elevated white blood cell count, abdominal exam often has positive psoas sign, positive obturator sign, positive Rovsing's sign, presence of guarding and rebound tenderness. Mesenteric ischemia presents in patients in their 70s and 80s who often have a history of atrial fibrillation. A clot lodges in the GI vessels causing abrupt severe abdominal pain. The abdominal exam in normal (i.e. no guarding or rebound tenderness) despite the patient reporting extreme pain (i.e. "pain out of proportion to the abdominal exam"). Cholecystitis typically presents in an obese female in her 40s or 50s with history of multiple pregnancies, RUQ abdominal pain worse after a fatty meal, some nausea and vomiting may be present, some low grade fever, positive Murphy's sign. Volvulus typically presents in older patients 70s, 80s, 90s, often nursing home or debilitated patients with a history of chronic constipation, presenting with progressively worsening abdominal pain (over hours), usually accompanied by abdominal distention; abdominal plain film radiographs often reveal "coffee bean sign", if barium enema is utilized then characteristic "bird's beak" or "ace of spades" finding from twisted cecum or sigmoid. Small bowel obstruction often presents in those with previous abdominal surgery secondary to adhesions, abdominal exam often reveals distention, pain on palpation (guarding, possibly rebound tenderness); auscultation reveals hyperactive bowel sounds with high pitched tinkling, metallic sounds with air rushes, with complete obstruction or bowel fatigue, patients may have periods of absent bowel sounds.

Which of the following agents is not thought to be associated with an increased risk of adverse cardiovascular events?

Aspirin (Bayer) 81mg given orally once daily is cardio-protective. All the other NSAIDs have a Black Box warning: -NSAIDs may increase risk of serious cardiovascular thrombotic events, myocardial infarction (MI), and stroke, which can be fatal -Risk may increase with duration of use -Patients with existing cardiovascular disease or risk factors for such disease may be at greater risk -NSAIDs are contraindicated for perioperative pain in setting of coronary artery bypass graft (CABG) surgery

A 63-year-old female with uncontrolled hypertension and a stroke is in the ED. Her BP has stabilized, and no other apparent cause was found for her stroke. EKG, bubble test, echocardiogram, and coagulation studies are all non-diagnostic. Now 5 days after the stroke, she is ready for discharge. What medications should be prescribed on discharge from the hospital.

Aspirin 81 mg daily Virtually all patients with an ischemic stroke need an antiplatelet medication, and aspirin 81 mg is the drug of choice. The others on this list, enoxaparin (Lovenox), warfarin (Coumadin), apixaban (Eliquis) are anticoagulants and should not be used in this setting. Anticoagulants pose the risk of causing hemorrhage within the stroke bed and should not be used. (The main exception to this is the patient who has a stroke caused by atrial fibrillation: These patients need anticoagulation.)

Among all the NSAIDs, aspirin is unique because it:

Aspirin irreversibly inhibits the COX-2 enzyme in platelets so they are totally ineffective for their 9 day life span. The platelet count will be normal but the platelet function analysis (PFA) will be abnormal.

A 5 year old female has a fever and muscle aches from a presumptive viral infection. Which one of the following medications would be the least appropriate analgesic agent for this patient?

Aspirin should not be used in children with symptoms of a viral infection because of Reye's syndrome. Acetaminophen and ibuprofen at the correct mg/kg dosing are considered safe. Heat and cold can be used for comfort also.

Which of the following is the most likely etiology for fever that occurs within the first 48 hours post-op?

Atelectasis is notorious as the etiology of fever at 36 to 72 hours post-op, and if untreated by incentive spirometry, early ambulation, deep breathing and coughing, will lead to pneumonia. Wound infection presents around 5-7 days post-op. Pulmonary embolism presents with shortness of breath, fatigue, chest pain, tachypnea, palpitations, usually several days after immobility, but should always be considered. UTI often presents with dysuria, urinary frequency 3-5 days after surgery or urethral catheterization. DVT often presents 5 to 7 days post-op and can present with lower extremity edema, erythema, tenderness, shortness of breath.

A 45 yo male presents to the clinic with a past history of hypertension diagnosed 15 years ago, but the patient stopped treatment after 1 year because of side effects. He has not had medical care for the past 14 years. His BP is 160/104 today. He has an elevated BUN and creatinine, proteinuria, and a low retic normocytic anemia. His diagnosis is

Chronic renal failure is the most likely diagnosis as a result of untreated hypertension. The elevated BUN/Creat and proteinuria demonstrate renal failure. The anemia is due to low erythropoeitin indicating chronic renal failure.

A 2 year old develops bloody diarrhea. He lives on a farm and often helps take care of chicks. Which of the following is the most likely cause of his illness?

Bacterial enteritis causes bloody diarrhea more commonly than viral enteritis - so this is unlikely a presentation of norovirus or rotavirus. The farm exposure to poultry makes salmonella a suspect. Salmonella is also found associated with eggs and reptiles. Other bacterial diarrheas and their prototypical sources include Yersinia from pork chitterlings, Campylobacter from birds, and Shigella from human fecal contamination.

A 52 year old woman presents to her primary care office with the skin appearance of "peau d' orange" of her right breast. Which of the following breast cancer types do you most suspect?

Because of its aggressive behavior and unusual appearance, Inflammatory breast cancer (IBC) can be easily misdiagnosed or treated inappropriately. One of the earliest changes associated with IBC is erythema, with the skin overlying the breast showing a pink or mottled pink hue. The color quickly changes to dark red or purple and spreads diffusely over the entire breast. The erythema may be associated with a sensation of heat in the affected breast. At about the same time, the breast begins to enlarge rapidly, sometimes increasing 2- to 3-fold in size in a period of a few weeks. This rapid rate of progression, along with the diffuse erythema that affects more than one third of the skin overlying the breast, helps distinguish IBC from locally advanced breast cancer with skin involvement. In about one third of patients with IBC, no discrete mass is palpable on clinical examination. From 55% to 85% of patients will present with metastases to the axillary or supraclavicular lymph nodes that may be detectable on clinical examination. Fixed palpable ipsilateral axillary nodes are a common finding. The increased size of the breast is the result of edema caused by tumor blockage of the lymphatic channels. The edema is associated with exaggerated hair follicle pits, causing a characteristic peau d'orange (orange peel) appearance to the skin. The blockage of the lymphatic channels can also cause wheals or ridging of the skin in the breast mound, and may be accompanied by a generalized induration of the breast. There may be nipple discharge; the nipple may appear inverted or flattened. In general, however, in disagreement with Sir Charles Bell's original description of IBC, these changes are not associated with pain. Paget's disease does sometimes have a rash associated but is more localized to the nipple. Ductal carcinoma in situ presents without a mass, but with calcification on mammography. Infiltrating ductal carcinoma often presents with a firm, irregular mass. Lobar carcinoma in situ presents with more of a thickening of tissue rather than a mass or skin changes.

A 64 year-old African American female with type 2 diabetes presents with blood glucose 650 mg/dL, elevated serum osmolality, and 3 days of nausea, malaise, headache, abdominal pain, vomiting, diarrhea, and polyuria. What is the first and most important initial goal of treatment?

Begin correcting her fluid and electrolyte depletion Marked hyperglycemia leads to osmotic diuresis, fluid and electrolyte losses, and dehydration. The first and most important initial treatment is fluid replacement. Fluid deficits can be as much as 6 liters in DKA and as much as 10 liters in HONC. A normal saline bolus is the first treatment, following by electrolyte replacement, insulin therapy, and correction of acidosis.

What is the most important risk factor for the development of type 2 diabetes?

Being overweight While all choices are risk factors for type 2 diabetes, the overwhelming risk of this disorder is related to being overweight or obese. Long ago, Hippocrates said, "When more food is taken than is proper, it occasions disease." He was right - over-nutrition, overweight, and obesity are the main drivers of the development of type 2 diabetes. While sedentary lifestyle, family history, ethnicity, and hypertension are also important associated risk factors, none is more important than the risk associated with being overweight. Thus, diet, exercise, and weight loss are the cornerstones of type 2 diabetes management.

In the treatment of anxiety disorder, which of the following has the highest potential for addiction?

Benzodiazepines are controlled substances with a huge potential for addiction. Both tolerance and withdrawal symptoms can occur. The onset of withdrawal typically happens within 2 or 3 days after the patient stops using it. However, with longer-acting drugs, such as diazepam, the latency before it begins to work can be 5 or 6 days. Side effects of benzodiazepines include sedation, confusion, memory deficits, respiratory depression, and addiction potential. Benzodiazepines with a slow onset of action and longer half-life or duration of action are the least likely to cause addiction in the patient. The following is a list of specific benzodiazepines and their specific indications: Lorazepam: used frequently in emergency situations because it can be given intramuscularly Clonazepam: may be used if addiction is a concern given it has a longer half-life Alprazolam: used frequently in panic disorder Chlordiazepoxide, Oxazepam: used frequently in treatment of alcohol withdrawal

A 22 year old female presents one day after a burn on the hand from hot water in a domestic kitchen setting. There is redness and painful blister formation over the dorsal hand. Which of the following is most correct with regard to the classification of this burn?

Blister formation is indicative of a second degree burn First-degree burns are usually red, dry, and painful. Burns initially termed first-degree are often actually superficial second-degree burns, with sloughing occurring the next day. Second-degree burns are often red, wet, and very painful. Their depth, ability to heal, and propensity to form hypertrophic scars vary enormously. Third-degree burns are generally leathery in consistency, dry, insensate, and waxy. These wounds will not heal, except by contraction and limited epithelial migration, with resulting hypertrophic and unstable cover. Burn blisters can overlie both second- and third-degree burns.

During a vaginal delivery, the head of the baby has emerged from the vaginal canal. The baby's head is retracting upward to the perineum. You are concerned about shoulder dystocia. What should be done next?

Brachial plexus injuries are one of the most important fetal complications of shoulder dystocia and one you want to avoid. When the head retracts against the perineum ("turtle sign") because the shoulder is stuck, you want to sharply flex the patient's legs into the abdomen and apply only suprapubic pressure to dislodge the shoulder. If the patient continues to push or fundal pressure is applied, this will lead to a brachial plexus injury. Never apply fundal pressure because that will only further trap the shoulder. A c-section should not be necessary if you reposition the patient, apply suprapubic pressure and do an episiotomy if needed for more room to facilitate delivery.

A 10-month-old infant is brought in by his mom for evaluation of a rash in his mouth. The infant is afebrile and appears happy and healthy. One week ago, he was successfully treated with antibiotics for otitis media. Examination of the oral epithelium reveals white curd like patches of the tongue and buccal mucosa which are easily scraped away. What is the most likely cause of this patient's condition?

Broad spectrum antibiotics and corticosteroids predispose patients to overgrowth of Candida albicans, causing the white patches of thrush on the lips, tongue, palate, and/or buccal mucosa. Incorrect answers: Staph aureus causes honey colored crusting and erosions of the skin. It is not associated with thrush. Strep pyogenes is commonly seen in strep pharyngitis and presents with fever, sore throat and tonsillar exudates and erythema. Herpes simplex presents as painful grouped vesicles on an erythematous base commonly noted on the face or lips of children.

An athlete has a knee injury, and the clinician performs a "Lachman Test" by pulling the tibia forward on the femur with the knee flexed. If the tibia translates anteriorly, i.e "gives" or moves anteriorly when the leg is pulled forward, what joint structure is most likely injured?

The Lachman tests for the integrity of the anterior cruciate ligament (ACL) Lateral collateral ligament (LCL) and medial collateral ligament (MCL) are tested with varus and valgus stress tests, respectively. The medial meniscus is tested via the McMurray Test and the posterior cruciate ligament is tested via the Sag Sign or posterior drawer test.

A 25-year-old male states he goes to all you can eat buffets and eats an enormous amount then feels guilty, induces vomiting, uses laxatives, and exercises excessively between big meals. He is 10% above normal BMI for his height and weight. The rest of his PE is normal and routine labs are normal. His diagnosis is:

Bulimia nervosa is described by frequent binge eating, as evidenced by eating large amounts of food in a distinct amount of time, as well as a lack of control of overeating episodes. This is shared with a compensatory behavior to prevent weight gain in the form of purging, misuse of laxatives and diuretics, fasting, and excessive exercise. Upon evaluation, the patient's body shape and weight are often the telltale factors that accompany this particular diagnosis. Anorexia nervosa is defined as the failure to maintain normal body weight, fear and preoccupation with weight gain, and body image disturbance. Amenorrhea is common from low body weight. These patients tend to deny their emaciated condition. They exhibit a significant concern with appearance and frequently examine and weigh themselves. These patients lose weight by strictly counting calories each day, excessive exercise, purging, and fasting, with laxative and diuretic use. It is seen more frequently in teenage girls between the ages of 14 and 18. Binge eating disorder is explained as compulsive overeating of large portions of food with an inability to stop or control it. It is illustrated by the individual engaging in binge eating episodes that last about 2 hours or on and off all day long. These people typically eat food even without being hungry and continue eating even after feeling pretty full. A new formal diagnosis is in the DSM-5 Obesity is characterized as a state of excessive body fat when the bodyweight surpasses 20 percent of the body mass index. Contributing factors of genetics and early development in childhood influence predisposition towards obesity. Obesity usually limits physical activity and, thus, exacerbates the situation.

Your patient is a 32-year-old depressed, obese male with a smoking history of 15 pack years. Which of the following antidepressant drugs is recommended for this patient?

Bupropion is an antidepressant that produces its effects primarily through the inhibition of the reuptake of norepinephrine, dopamine, and serotonin. It has increased risk for seizures, but its side effects include weight loss. Thus, it is an ideal medication to use in a patient who is overweight. An additional advantage of using Bupropion is that it reduces cravings and other withdrawal effects in those individuals wanting to quit smoking. Fluoxetine is an SSRI used in the treatment of depression. It may take 4-6 weeks before a beneficial effect is noticed. Side effects from Fluoxetine include headaches, weight changes, sexual side effects, and GI disturbances. Escitalopram is an SSRI also used in the treatment of depression. The side effect profile is similar to Fluoxetine. Mirtazapine is a tetracyclic antidepressant that also has anxiolytic and appetite-stimulating properties. Therefore, weight gain is a potential side effect, so this drug should not be given to overweight patients.

You have a 35 year old male patient with depression. Your patient expresses concern about sexual dysfunction. Which of the following drugs used for the treatment of depression has the least sexual dysfunction side effects?

Bupropion is first-line antidepressant for those individuals wishing to quit smoking and who may be experiencing a loss of sexual function. The other drugs are SSRIs. They are notorious for causing sexual dysfunction and should be given cautiously to those patients already depressed and experiencing decreased libido.

A 42-year-old male with schizophrenia had side effects with his first dopamine receptor agonist medication. His psychiatrist prescribed Clozapine (Clozaril). What follow-up is suggested every week?

Clozapine has been associated with few, if any, extrapyramidal side effects or tardive dyskinesia. It is an appropriate treatment for schizophrenic patients that have not responded to first-line dopamine receptor antagonists or who have tardive dyskinesia but is not a first-line drug for treating schizophrenia. It is associated with 1 to 2 percent, incidence of agranulocytosis and thus requires weekly CBC monitoring looking for low WBC especially a low absolute neutrophil count (ANC) Before initiating, baseline ANC must be ≥1500/mm3 for the general population and ≥1000/mm3 for patients with documented benign ethnic neutropenia. Regularly monitor ANC during treatment. Advise patients to report symptoms consistent with severe neutropenia or infection immediately (e.g., fever, weakness, lethargy, sore throat)

A 35 year old man has had numerous episodes of severe headaches for the past two weeks, with severe pain behind the right eye, and overproduction of tears as if crying with one eye. Each episode lasts less than an hour. His neurological exam is normal. This presentation is most consistent with what diagnosis?

Cluster headache The cluster headache is "enough to make a grown man cry," often occurring in the morning hours, with numerous headaches of short duration, with unilateral retro-orbital pain and profuse lacrimation.Temporal arteritis is a headache with scalp tenderness and elevated sedimentation rate. Tension headache is the most common benign headache characterized by bilateral pressure pain, worse toward the end of the day. Migraine headaches are usually unilateral throbbing headaches with photophobia and phonophobia.

You are working on the ortho consult service at the local trauma hospital, and one of the floor nurses calls you regarding a patient who has been admitted and is going to the operating room tomorrow morning for open reduction internal fixation of a tibia fracture. She states that he is writhing in pain, even after maxing out his pain medication, and begging her to loosen his splint. She asks if she can take the ACE wrap off to make it a less tight. You tell her yes. She calls back in 15 minutes and states that the patient is still crying in pain and that he is now complaining that he can't feel his toes. What are you most worried about?

Compartment syndrome: pain out of proportion to the clinical situation is the hallmark sign of compartment syndrome (along with paresthesias, pallor, paralysis, pulselessness, pressure). This is a surgical emergency and should be addressed immediately

Your 55-year-old African American female patient presents to your primary care clinic complaining of sudden new onset low back pain. She denies any injury or trauma to her back. Her review of systems is also positive for malaise, weakness, and nose bleeds. What pathognomonic finding would you suspect to see on her lumbar spine x-rays?

Lytic lesions: her demographics and subjective symptoms should make you suspicious for multiple myeloma. Lytic lesions are the hallmark sign for this malignancy, and are often found in the axial skeleton.

Which analgesic medication has aspects of opioid antagonist and opioid antagonist features like naloxone?

Nalbuphine (Nubain) is a synthetic opioid agonist-antagonist potent analgesic. It stimulates kappa opioid receptor in the CNS, which causes inhibition of ascending pain pathways. It is indicated for the relief of moderate to severe pain. It protects respiratory drive and is useful for preventing itching. It is not scheduled by the DEA

Your colleague in the emergency room is assisting with the treatment of an anterior knee dislocation. He performed an ankle brachial index (ABI) prior to the knee reduction that was abnormal (<0.9) and then performed another ABI after reduction that was within normal limits (>0.9). Your colleague was concerned about the integrity of which structure? A. Posterior tibial artery B. DP artery C. Popliteal artery D. Deep femoral artery

C. Popliteal artery A. Posterior tibial artery: this artery's pulse should be palpated to make sure it is present and normal (compare to contralateral side). However, this does not guarantee the absence of a popliteal artery injury and should only be used to raise your clinical suspicion for a popliteal artery if the pulse is diminished B. Dorsalis pedis artery: this artery's pulse should be palpated to make sure it is present and normal (compare to contralateral side). However, this does not guarantee the absence of a popliteal artery injury and should only be used to raise your clinical suspicion for a popliteal artery if the pulse is diminished C. Popliteal artery: this artery is commonly injured in knee dislocations. An injury to the popliteal artery can be catastrophic and lead to amputation if not diagnosed and addressed within 8 hours (80%+ amputation rate) D. Deep femoral artery: this artery is proximal to the knee and unlikely to be injured in a knee dislocation

A 55 year old man with atrial fibrillation presents with complaint of acute visual changes within the past day, "like looking through a glass of water." He denies eye trauma or headache. On exam his visual acuity is normal, and pupils are 3mm and reactive to light. Extraocular movements are intact. However on visual fields testing by direct confrontation he has a field deficit of the left visual field of each eye. The patient is transferred from the primary care office to the emergency department. What is the most important initial diagnostic test to workup this patient's visual disturbance?

CT non-con You recognize that the visual problem reflects a neurological problem, not a problem of the eye itself, and so you order a CT of the head without contrast. On arrival to the ER, any patient with new onset of neurological symptoms suggestive of stroke should go immediately for a CT scan of the head. The purpose of the CT is to look for structural problems of the brain such as hemorrhage or mass. Do not be surprised if the CT comes back normal. An ischemic stroke presents with an initially normal CT of the head. The normal CT does not mean there is no stroke, it just means it is not a hemorrhage or tumor. Skull x-ray can be useful for the evaluation of possible radiopaque foreign body but has little other utility. MRI of the brain can be a second line test to evaluate for ischemic stroke, but the CT should be done first because of its universal availability and its ability to rule out hemorrhage. Slit-lamp exam is useful to evaluate eye trauma like corneal abrasion or herpes keratitis.

Which of the following statements about vaginitis is true?

Candidiasis can have a "cottage-cheese" like discharge, produces intense vulvovaginal pruritis, and is treated with -azoles, but it will have a negative whiff test. Bacterial vaginosis is not a sexually transmitted infection; however, new/multiple sexual partners can cause an imbalance that leads to BV. It does cause a thin, grey discharge, but it typically has a "fishy" odor and increases the normal vaginal pH. Atrophic vaginitis is seen in postmenopausal women and is treated with vaginal estrogen.

A 25 year old female patient presents with a complaint of vaginal discharge for two days. She is afebrile and does not have abdominal pain. Speculum exam reveals an edematous and inflamed cervix with a mucopurulent discharge. Wet prep is negative for clue cells, flagellated organisms, and hyphae. Which of the following is the best treatment?

Ceftriaxone 250mg IM is the CDC recommended current first-line treatment choice for Neisseria gonorrhoeae. Gonorrhea can be asymptomatic or associated with a mucopurulent discharge. CDC recommends concurrently treating for chlamydia with 1 g oral azithromycin. Dicloxacillin is typically used to treat staph infections. Fluoroquinolones are no longer indicated in the treatment of gonorrhea because they have shown resistance. Metronidazole is indicated to treat bacterial vaginosis infections or trichomoniasis.

An 18 year old male college student presents to the ER with 1 day history of fever, headache, and a stiff neck. He is suspected of having bacterial meningitis. You want to start drug treatment as soon as possible. What empiric therapy will you begin?

Ceftriaxone and vancomycin In this 18-year-old the most likely pathogens are strep pneumoniae and meningococci, and so you want a third generation cephalosporin and vancomycin.The treatment of antibiotic associated colitis, pseudomembranous colitis, C. difficile can be either oral metronidazole or oral vancomycin. Treatment of toxoplasmosis includes pyrimethamine plus sulfadiazine plus leucovorin. Treatment of methicillin-resistant staph aureus includes trimethoprim-sulfamethoxazole or doxycycline as first line agents, and vancomycin IV in more severe cases.

A farmer intentionally ingests malathion (a pesticide) in a suicide attempt. He presents with a classic cholinergic toxidrome. Which of the following clinical features would you expect to find?

Cholinergic toxidrome= BAD SLUDGE (bradycardia, anxiety, delirium, salivation, lacrimation, urination, defecation/diarrhea, GI distress, emesis) or Killer Bs (bradycardia, bronchorrhea, bronchospasm) and SLUDGE. versus Anticholinergic= "Hot as a Hades, Red as a beet, Blind as a bat, Dry as a bone, Mad as a hatter" which is hyperthermia, flushed dry skin and mucus membranes, diplopia, urinary retention, Altered Mental Status

A 30 yo female presents to the ED with acute dyspnea and right sided pleuritic chest pain. Her ABG: pH 7.3, pCO2 50, pO2 80, HCO3 24 Normal values pH 7.0-----------7.4-------------7.6 PCO2 >60--------45—35----------<20 HCO3- <18--------22—26--------- >32 Your patient has:

Respiratory Acidosis from a suspected Pulmonary Embolus

A 60 yo male with COPD presents to the ED with 3 days increasing dyspnea and a dry cough. His ABG: pH 7.33, pCO2 50, pO2 80, HCO3 30 Normal values pH 7.0-----------7.4-------------7.6 PCO2 >60--------45—35----------<20 HCO3- <18--------22—26--------- >32 Your patient has:

Respiratory Acidosis with his COPD, with renal compensation. He has an acidosis explained by the elevated CO2 (respiratory) but renal compensation has occurred with the elevated bicarb

A 71-year-old man presents with a new growth of the left jaw for 6 months. He is a Florida native, who grew up on a farm, and admits to frequent sunburns and minimal sunscreen use. He states the lesion on his jaw is growing and bleeds easily when he washes his face. Upon examination of the skin, he is fair skinned with extensive photodamage. On the left side of the jaw, there is a 4mm pearly papule with telangiectasias, rolled borders and a central erosion. Given these findings, which of the following is the most likely diagnosis?

Correct Answer is Basal Cell Carcinoma (BCC), the most common sun related skin cancer. Clinical diagnosis of BCC is characterized as a pearly, pink papule with telangiectasias and rolled borders noted on an area of sun exposed skin in patients with poor tanning capability and signs of photoaging. Incorrect Answers: Malignant Melanoma presents clinically as an ill-defined, asymmetric, irregularly pigmented dark macule or papule. Squamous Cell Carcinoma presents clinically as an indurated, thick, scaly plaque or nodule noted over sun exposed areas. Actinic Keratosis is a precancerous lesion that presents as a rough, adherent, scaly papule over sun exposed areas.

This type of skin cancer is related to UVA light exposure, may develop for years prior to detection, and is the most common cancer of adults age 25-29: [Picture is dark macule, some parts darker, irregular border]

Correct answer is malignant melenoma. Early melanomas may be differentiated from benign nevi by the ABCDs, as follows: A - Asymmetry B - Border irregularity C - Color that tends to be very dark black or blue and variable D - Diameter ≥6 mm An actinic keratosis is a potential pre-cancer from excessive sun damage but not a skin cancer. BCC and SCC are more commonly seen in those over age 40.

A 28 year old male presents to clinic for routine followup. He and his wife are wanting to start a family this year. He has a history of right sided cryptorchidism as a child which was surgically corrected. What should you counsel patient on?

Cryptorchidism increases a male's chances of infertility and testicular cancer in the future. Successful scrotal repositioning of the testis may reduce but does not prevent the potential long-term issues of infertility and testis cancer. You should recommend frequent testicular exams with or without scrotal U/S and tumor markers, and semen analysis if the couple can not conceive.

You are doing a routine exam on a 17 year old female for her high school sports program. She asks when should she begin having pap smears?

Current recommendations state that pap smears should begin at age 21. The Pap test is indicated to screen for malignant and premalignant lesions of the cervix. The recommended age at initiation of cervical cancer screening has undergone significant revision over time as the natural history of HPV infection and subsequent cervical dysplasia has been elucidated. Although former guidelines recommended starting Pap smear screening at age 18 or the onset of sexual activity, these guidelines were revised in 2006 to recommend initiation 3 years after the onset of sexual activity or age 21, whichever comes first. In 2009, these were further revised to recommend that cervical cancer screening begin at age 21, regardless of sexual history. This recommendation was confirmed in 2012 by ACOG and again in January 2016.

Which of the following is true about developmental hip dysplasia?

Developmental hip dysplasia may be present at birth, or it may become apparent later - it is not necessarily congenital. It's more common in girls, and it's more common in first born children especially when born prematurely or from a breech position. If caught early, a simple harness can be curative. Plain xrays may not be diagnostic if used earlier than 6 months; earlier exams should be done via ultrasound.

A 30 yo male presents to the ED with acute anxiety, dyspnea, chest pain, and tingling sensation in his hands. His ABG is pH 7.5, pCO2 30, pO2 100, HCO3 24 Normal values pH 7.0-----------7.4-------------7.6 PCO2 >60--------45—35----------<20 HCO3- <18--------22—26--------- >32 Your patient has:

Respiratory Alkalosis: This patient has the classic signs of hyperventilation syndrome. One must rule out other acute causes such as PE and pneumothorax

Anorexia nervosa is defined as a refusal or inability to maintain weight at (or above) what percent of ideal body weight?

Diagnostic criteria for anorexia nervosa: 1. Refusal to maintain weight at or above the normal weight for one's age and height (85% of expected weight caused by weight loss or lack of expected weight gain) 2. Prominent fear of gaining weight or becoming fat despite being underweight 3. Disturbance in the way one's body weight or shape is experienced, undue influence of body weight or shape on self-evaluation, or denial of the severity of the current low body weight 4. In females who are beyond menarche, amenorrhea may be present Diagnosing anorexia nervosa can sometimes be a challenge. Adolescents are usually quite adept at hiding the symptoms. Patients should be weighed routinely in a physician's office, and any substantial weight loss should be assessed with anorexia nervosa in the differential diagnosis. Abnormalities in blood chemistry studies and ECG changes can serve as evidence of anorexia when present.

A 61 year old man presents with a soft mass located at the external inguinal ring. He states that in the morning it is flat and at the end of the day it pouches outward and mildly aches. On physical examination, you feel the mass pressing on the side of your examining finger during the Valsalva maneuver when your finger is in the inguinal canal. Which of the following is most likely?

Direct hernia is classically described. Femoral hernia presents with pain in the "empty space with lymphatics", remember: NAVEL= "nerve, artery, vein, empty space with lymphatics" as you go from lateral to medial in the femoral triangle. Umbilical hernias are near the umbilicus. On physical examination, with indirect hernias, you feel the mass pressing on the TIP of your examining finger during the Valsalva maneuver when your examining finger is in the inguinal canal. They follow the path of the descending testicle and may go into the scrotum, often younger males and no pouch/swelling is associated at the external ring. They are more common than indirect hernia. Ventral hernia is often located in the midline of the abdomen and associated with multiple pregnancies or abdominal surgery.

A 22-year-old female presents with a complaint of a "mass" in her right breast that she has noticed for about 3-4 months. It is painless, moveable, and doesn't come and go with her menstrual cycle. On exam, a 5 cm, freely movable, firm mass is palpated in the right breast without discomfort. What is the most likely diagnosis?

Fibroadenoma is the most common breast tumor in menstruating women under the age of 25. Typically, it is a unilateral, firm, freely moveable, slow growing, painless lump in a breast. The management is an excisional biopsy. A breast abscess would be painful and cause redness and swelling to the affected area. Breast cysts are fluid filled and tend to fluctuate in size and tenderness with menstrual cycles. Fibrocystic changes tend to be areas of thickening or lumpiness that blend into surrounding tissue and don't have delineated borders. They fluctuate, as well, with menstrual cycles and can be bilateral, as can breast cysts.

Which of the following is typically not seen in fibrocystic breast changes?

Fibrocystic changes are associated with the menstrual cycle and, therefore, have a cyclic pattern, not persistent pain. Fibrocystic breasts can be tender or painful, but it usually occurs somewhere from ovulation to the beginning of menses and does not remain persistent.

72 year old male with 3 episodes of nocturia, weakened urinary stream and urinary frequency throughout the day. You diagnose BPH (benign prostatic hyperplasia). He is interested in oral medication. What do you start him on?

Flomax is an alpha-1 blocker that relaxes the smooth muscle of the bladder neck and prostate thereby allowing better urine flow. Patients that respond to this medication will experience a stronger stream and decreased urinary frequency. Oxybutynin is used for Overactive Bladder. Finasteride is a 5-alpha reductase inhibitor that should be used second line for BPH. Cranberry Tablets can be useful for females and males with recurrent UTIs but not in this case.

A 15 year old male presents to the clinic with 1 month increasing dyspnea on exertion and fatigue. On physical examination you notice his sclera are jaundiced and his palpebral conjunctiva are pale. What is the most likely cause of these findings?

Jaundice and pallor of the conjunctiva are signs of hemolytic anemia. The jaundice is caused by the increased breakdown of hemoglobin into indirect bilirubin that must be delivered to the the liver to be conjugated into direct bilirubin. A CMP should be obtained to quantify the indirect, direct and total bilirubin. High indirect bilirubin, with an elevated LDH and a falling Hb/Hct indicates hemolysis. Hepatitis and Cholelithiasis would give an elevation of both indirect and direct bilirubin and elevate AST/ALT transaminases.

Familial hyperlipidemia causes which set of lipid abnormalities?

Elevated total cholesterol and LDL Familial hyperlipidemia causes severe elevation in total cholesterol and LDL, but not triglycerides. Signs and symptoms tend to be consistent with early-onset ischemic heart disease. Homozygous familial hyperlipidemia is rare, occurring about 1:1,000,000 and most patients do not survive beyond 30 years of age. Heterozygous is more common (1:300) but less severe.

Which type 2 diabetes treatment is most beneficial for a 63 year-old male with a history of coronary artery disease, congestive heart failure, and history of myocardial infarction 3 years prior with subsequent percutaneous transluminal coronary angioplasty (PCTA)?

Empagliflozin The sodium glucose cotransporter-2 class of diabetic medications has shown decreased rates of all-cause and cardiovascular-related death (empagliflozin), as well as lower rates of CHF-related hospitalization, decreased rates of death, myocardial infarction and non-fatal stroke (canagliflozin).

A 42-year-old female patient presents for an annual exam. When asked about her menstrual cycles, she states that they are still occurring monthly, but they have become very heavy and prolonged. Her pelvic exam was unremarkable. What should be done next?

Endometrial hyperplasia and endometrial cancer must be ruled out in this patient because of her age and changes in her cycles. An endometrial biopsy can evaluate for both. A hysterosalpingography evaluates the patency of the fallopian tubes. A hysteroscopy is the next step after doing an endometrial biopsy that is negative and the patient has continued heavy, prolonged bleeding. A colposcopy evaluates the cervix.

Which of the following is NOT a common result from endometriosis?

Endometriosis does not result in amenorrhea. Typically, endometriosis can impact cycles by making them last longer and have fewer days between the cycles. Symptoms incude: Dysmenorrhea, heavy or irregular bleeding, pelvic pain, lower abdominal or back pain, and dyspareunia. pain on defecation - often with cycles of diarrhea and constipation, bloating, nausea, and vomiting, pain on micturition and/or urinary frequency, and pain during exercise.

Which is not a risk factor for developing a hernia?

Excessive lifting/ exercise/ trauma, obesity, pregnancy, COPD, BPH, straining to defecate, and smoking are all risk factors for developing a hernia. Having a previous MI is not a risk factor.

Tennis elbow is associated with pain at the lateral epicondyle caused by repetitive:

Extension/ abduction of the hand at the wrist is correct. The ERCB( Extensor Carpi Radialis Brevis) originates at the lateral condyle of humerus and inserts into the base of 3rd metacarpal and is in direct relationship with the ECRL, which inserts into the base of the 3rd metacarpal which allows for extension and abduction of the hand at the wrist. The ECRB is prime dorsiflexor (extender) of the hand.The other choices do not describe the function of the ERCB which is the primary issue with lateral epicondylitis.

A patient with frequent stools, heat intolerance, warm moist skin and tremor is most likely to have which of the following laboratory findings:

Hyperthyroidism causes high T4 and T3 levels because of overproduction of the hormones by the thyroid gland. Because of negative feedback to the pituitary, hyperthyroidism is associated with low TSH.

A 38 year old man presents with muscle cramps and on PE a positive Chvostek's sign (facial spasm when tapping on the cheek over the facial nerve). He has a prolonged QT interval on EKG. What is the electrolyte abnormality

Hypocalcemia is manifested by lengthening of the ST segment and eventually in a prolonged QT interval. Hypomagnesaemia primarily cause arrhythmias.

After 3 months of high-intensity statin therapy, your patient's fasting triglyceride levels remain approximately 500-600 mg/dL (although they have fallen from 1000-1200 mg/dL). What class of medications is the best choice for the next step in therapy?

Fibrates The first step in the lowering of triglyceride levels is to reach the LDL goal with statins. If triglyceride levels remain elevated, the best category of medications for lowering triglyceride level is the fibrates, although other categories, such as omega-3's, can also lower TG. In addition to medication, low fat diet, exercise, weight reduction, smoking cessation, and limiting alcohol intake are also effective for TG reduction. There is an increased risk of pancreatitis with TG levels over 500-1000, which is an additional reason for lowering levels.

A 71 year old woman presents with a history of vomiting and diarrhea for the past several days secondary to a viral illness. She is afebrile, heart rate is 120/min, respirations 25/min, BP 100/60 mmHg. She is pale and lethargic. Which of the following is most likely and what is the initial treatment of choice?

GI losses can lead to severe dehydration/loss of volume which causes tachycardia, hypotension, pale skin, oliguria- she needs IV fluids

Which of the following are NOT decreased in recipients of HPV vaccinations?

HPV vaccination prevents both genital warts and cancers that are triggered by HPV infections, such as cancers of the head and neck, cervix, penis, and anus. It does not prevent Hepatitis B or other STDs

What is NOT a cause of pre renal acute kidney injury (AKI)?

Hemolysis may cause intra-renal acute kidney injury secondary to excessive free hemoglobin flowing through the kidney. The other choices all decrease kidney perfusion and cause pre renal AKI.

A 23-year-old female is diagnosed with high-grade squamous intraepithelial lesion (HSIL) after a routine Pap smear. Which of the following is the most appropriate next step?

High-grade squamous intraepithelial lesion - HSIL- indicates moderate/severe dysplasia. There is an increased risk the abnormal cells to develop into cancer. A colposcopy is the next step to confirm pap results. If a cervicitis was present, you can treat with antibiotics and repeat the pap. If the pap showed an ASCUS result, a repeat pap in 6 months can be scheduled. A LEEP is a procedure that could be done after confirming HSIL by colposcopy. An ablative procedure is not an appropriate first step because ablative procedures do not provide a specimen for diagnostic evaluation.

A 13-year-old overweight African-American male presents to your general pediatrics clinic with his mother. She tells you that he has been complaining of knee pain for the past 2 weeks. He denies any injury or trauma to his knee, and his knee exam is benign. However, his gait is markedly antalgic. What other joint should you immediately examine due to the possibility of an injury requiring urgent surgical intervention?

Hip: slipped capital femoral epiphysis is commonly seen in obese adolescent African-American males and can often present as knee pain due to the compression of the medial obturator nerve. SCFE can lead to AVN and thus should be evaluated by an orthopedic surgeon urgently.

When counseling your 56 year old male patient about oral medication for Erectile Dysfunction you list the side effects of Sildenafil. Which of the following is NOT a side effect of this medication?

Hypertension is not a side effect of Sildenafil (Viagra). Hypotension is a side effect which is why some patients may experience dizziness. Other choices are typical side effects of the medication

65-year-old man underwent successful left knee replacement surgery yesterday and was receiving IV hydration for a second day. He was very thirsty and drank several containers of water. His wife reported he developed new confusion and disorientation. Physical examination revealed disorientation to place and time, but the rest of the PE was unremarkable, including normal vital signs and pulse oximetry. EKG was also performed and did not show any abnormalities in comparison to preoperative EKG. What electrolyte finding is most likely causing this?

Hyponatremia severe enough to produce confusion and nausea and vomiting can occur with IV overhydration with hypotonic fluids Hypercalcemia may report non-specific symptoms, such as constipation, fatigue, and depression. Although the nausea and confusion experienced by this patient can be caused by hypercalcemia, his EKG would show a shortened QT interval The hypernatremic patient is typically hypovolemic due to free water losses. Hyperthermia, delirium, seizures, and coma may be seen with severe hypernatremia, i.e., sodium >158 mEq/L. Hyperkalemia and hypokalemia are unlikely in this patient, as he is not experiencing muscular weakness, fatigue, cramps, ileus, hyporeflexia, or paralysis, which can be seen in these conditions. The EKG was also normal and did not reflect the T wave changes seen with abnormal potassium levels.

A 5-year-old boy has 2 days of excessive nosebleeds. He had a viral illness 2 weeks ago, with fever and sore throat, that resolved, and now he is active, afebrile, and seems healthy to the parents. The CBC reveals normal hemoglobin, hematocrit, and WBC count but a low platelet count of 40,000 (normal 200,000 to 400,000). The PT and aPTT are normal. What is the most likely diagnosis?

ITP is usually a spontaneously resolving thrombocytopenia appearing after a viral illness in children. It is typically benign and 80% of cases in children resolve in a few weeks. 20% of ITP can become chronic and need hematology care. TTP would present with evidence of multi-organ failure. HUS would present with acute renal and hepatic failure with red cell hemolysis. Henoch-Schönlein purpura would present with palpable purpura. ITP in children should be managed with observation and no contact sports. First line treatment, if needed, is steroids.

Which substance when abused can cause danger in overdose, danger in withdrawal, and physical dependence?

In order to understand the above question, knowledge of specific definitions of intoxication, withdrawal, abuse, and dependence must be understood. First, intoxication refers to a reversible experience with a substance that leads to either psychological or physiological changes. Secondly, withdrawal means that cessation or reduction of a substance leads to either psychological or physiological changes. Third, abuse refers to maladaptive pattern of use of substances that leads to engaging in hazardous situations, legal problems, inability to fulfill obligations, and continued use despite adverse consequences. And, lastly, dependence is a maladaptive pattern of use of substances that leads to tolerance. There is withdrawal when trying to cut down. Patients spend a great deal of time engaging in drug use. There is continued use despite adverse consequences. Alcohol is the one substance, when abused, which can cause danger in overdose, withdrawal, and physical dependence. Patients with alcohol withdrawal develop signs and symptoms of withdrawal between 12-48 hours after the last drink. During the acute stage they develop sweating, hyperreflexia, seizures, and tremors. This is followed by alcoholic hallucinosis (auditory and visual) in the absence of autonomic symptoms. When alcohol intoxication approaches the danger of overdose, mechanical ventilation is sometimes required if symptoms are severe enough. Additionally, the signs and symptoms of withdrawal can reach a dangerous high when the patient begins to experience tremors, hallucinations, seizures, and delirium tremens. Delirium tremens (DTs) is a dangerous state of alcohol withdrawal that sends the autonomic system into overdrive, which can progress to cardiovascular collapse. Altered mental status is also seen in this condition. DT's is, indeed, a medical emergency with a high mortality rate. Cocaine withdrawal is not as fatal as alcohol withdrawal, although, the patient can still experience deleterious effects. Such effects include: anxiety, tremulousness, headache, increased appetite, and depression. Amphetamine withdrawal is similar in nature to cocaine withdrawal. The withdrawal from amphetamines is not as life threatening as alcohol withdrawal. In overdose, patients become euphoric, hyperactive, experience perceptual disturbances, and sustain weight loss. Opiate withdrawal is often very trying on a patient, but has less risk of death when compared to alcohol withdrawal. Patients often experience fever, chills, lacrimation, abdominal cramps, muscle spasms, and diarrhea.

A 27-year-old male patient presents with delusions, hallucinations, disorganized behavior and speech for the last eight months. These symptoms are causing significant disruption in his social and occupational functioning. What is your diagnosis?

In this scenario, the patient demonstrates hallucinations, delusions, disorganized speech, and disorderly behavior over at least six months; thus, he meets the criteria required for a diagnosis of schizophrenia. An antisocial personality disorder is described by a continuing pattern of behavior in which the basic rules of society and the rights of others are violated. It is seen in adults' ages 18 or older, and these people are common criminals in and out of the jail system. Children with conduct disorder often develop an antisocial personality disorder. Patients with borderline personality disorder exhibit a pattern of volatility in relationships and prominent impulsivity. They fluctuate between deriding and admiring people (with other people viewed entirely good or entirely bad, a phenomenon is known as "splitting"). These individuals tend to be reckless and irresponsible and may illustrate suicidal or self-mutilating behavior. Common emotions conveyed are anger and long-standing emptiness. The DSM-5 characterizes dysthymia (persistent depressive disorder) as a depressed mood lasting most of the day, for the majority of days, for at least two years. The diagnosis also requires at least two of the following symptoms: decreased or increased appetite, insomnia or hypersomnia, poor energy, low self-esteem, feelings of worthlessness, and impaired concentration. Patients with dysthymia often say that they have felt depressed their entire life. Dysthymic patients' symptoms are similar to those with a major depressive disorder but are less harsh and present over a more extended period.

Your patient is a 39 year-old female with an extensive family history of type 2 diabetes. Her BMI is 45 and she is on basal-bolus insulin therapy as follows: Insulin glargine 37 units at bedtime daily, and Insulin lispro 8 units with each meal. Her home glucose log shows an average morning fasting blood sugar of 114, average pre-lunch glucose of 139, average pre-dinner glucose of 169, and average bedtime glucose of 137. What is the best adjustment to make to her insulin regimen?

Increase her lunch-time dose of short-acting insulin Given that her pre-dinner average glucose readings are the highest of the day, the most appropriate adjustment would be to increase her lunchtime short-acting insulin to help better control her afternoon and early evening glucose levels. While we could also consider instituting sliding scale models of insulin administration or carbohydrate counting, a good starting place would be to adjust her lunchtime insulin and revisit diabetic diet and meal composition.

Which of the following agents should be reserved for the treatment of gouty arthritis because of the risk of central nervous system and renal toxicities?

Indomethacin is reserved for acute gout attacke because the main side effects are: Transient renal insufficiency (40%) Jaundice (≤15%) Elevated liver function test values (≤15%) Headache (12%)

Your 22-year-old patient presents to your primary care clinic for evaluation of wrist pain after a fall on an outstretched hand. She complains of tenderness in the "snuffbox" region as you palpate. Her x-rays are negative and do not reveal a fracture. Which of the following would be the least appropriate action to take? A. Provide adequate pain control in the form of oral medications, possibly a short course of prescription/opioid: this is appropriate fracture management B. Place her in a thumb spica splint and refer to ortho: this is the appropriate split for a suspected scaphoid injury C. Inform her that her x-rays are negative and that she likely suffered a bone contusion: scaphoid fractures commonly lag on x-ray. These fractures can lead to AVN if not treated, so it is imperative that repeat x-rays are obtained about 2 weeks after injury if clinical suspicion is high D. Have her follow up with ortho in 14 days for repeat x-rays: this is an appropriate referral if your clinical suspicion for a scaphoid fracture is high.

Inform her that her x-rays are negative and that she likely suffered a bone contusion: scaphoid fractures commonly lag on x-ray. These fractures can lead to AVN if not treated, so it is imperative that repeat x-rays are obtained about 2 weeks after injury if clinical suspicion is high

Which of the following medications is the most appropriate initial treatment for type 1 diabetes mellitus?

Insulin Type 1 diabetes is characterized by autoimmune beta cell destruction with an absolute insulin deficiency, rather than beta cell dysfunction with a relative insulin deficiency as in type 2. While there are a variety of treatment modalities for type 2 diabetes (decreasing hepatic production of blood glucose, increasing tissue sensitivity to glucose uptake, secreting more insulin from the pancreas, increasing urinary excretion of glucose, etc), there is only one treatment for type 1 diabetes - insulin.

A 43 year-old male presents with progressive fatigue, blurry vision, polyuria, and polydipsia over the last week. He has no past medical or family history of diabetes. His urine is positive for 2+ glucosuria, but no ketones. His blood glucose check is 317 and A1C is 11.4. What is the initial treatment of choice for this patient?

Insulin glargine and aspart This patient can be diagnosed with diabetes on the basis of his random blood glucose because it is very high and is accompanied with classic symptoms of diabetes. Although diabetic diet and exercise are the cornerstones of diabetic management, he should be started on insulin because his presenting blood glucose is over 300 and A1C is over 10. Although he might be able to transition back to oral agents and discontinue insulin, he should start insulin immediately to most effectively regulate his blood glucose levels.

You suspect a right sided varicocele in your 36 year male patient as the cause of his 1 day scrotal swelling. What should be your next step for this patient?

It is very important to obtain a CT Abd/Pelvis in this patient. If a patient has sudden onset of a varicocele, a single right-sided varicocele, or any varicocele that is not reducible in the supine position, consider possible retroperitoneal pathology (eg, renal cell carcinoma) as the cause of spermatic vein compression. Investigate further with appropriate ultrasonography or CT scanning before repairing the varicocele. However, obtaining an U/S after the CT and discussing infertility would also be next steps that you can have with the patient at that office visit.

Which of the following is not true of osteoarthritis? A. It is the most common form of joint disease and affects more than 30 million Americans B. It is the leading cause of chronic disability in older adults C. It most commonly found in non-weightbearing joints, specifically those in the hands D. First-line treatments include weight loss, exercise, physical therapy, and NSAIDs

It most commonly found in non-weightbearing joints, specifically those in the hands; False - OA is most commonly found in the weight-bearing joints of the body (ex. Hip, knee, ankle) A. It is the most common form of joint disease and affects more than 30 million Americans - True B. It is the leading cause of chronic disability in older adults - True C. It most commonly found in non-weightbearing joints, specifically those in the hands; False - OA is most commonly found in the weight-bearing joints of the body (ex. Hip, knee, ankle) D. First-line treatments include weight loss, exercise, physical therapy, and NSAIDs - True

A 9-year-old girl is seen in the clinic with complaints of a left sided limp. The mother claims that the child has been crying and has been anorexic, lethargic and febrile for the past 13 weeks. She denies any trauma but also noticed that she started to limp recently. She has missed school for the past 2 weeks. The physical exam reveals an ill-looking child with a temperature of 101F, swollen right knee and ankle, and there is marked joint stiffness. Her muscles in the leg are tender to deep palpation. Abdominal exam reveals hepatosplenomegaly and linear salmon-colored rash on the trunk. Blood work reveals elevation of CRP, and positive ANA antinuclear antibodies. Her leg x-rays are unremarkable except for the soft tissue swelling. Which of the following is the diagnosis?

Juvenile Rheumatoid Arthritis/ Juvenile Idiopathic Arthritis (JIA) Diagnosis History findings in children with JIA may include the following: Arthritis present for at least 6 weeks before diagnosis (mandatory for diagnosis of JIA) Either insidious or abrupt disease onset, often with morning stiffness or gelling phenomenon and arthralgia during the day Complaints of joint pain or abnormal joint use History of school absences or limited ability to participate in physical education classes Spiking fevers occurring once or twice each day at about the same time of day Evanescent rash on the trunk and extremities Psoriasis or more subtle dermatologic manifestations

The main advantage of ketorolac over aspirin for pain management is:

Ketoralac Is available in a parenteral formulation that can be injected intramuscularly or intravenously. That gives it an advantage over all the oral NSAIDS in the acute pain setting. Ketoralac is a NSAID with all the same side effects and warnings.

Which of these vaccines would be contraindicated in a child with Severe Combined Immunodeficiency Syndrome?

Live vaccines are contraindicated in children with immunodeficiencies such as SCID. The live vaccines on the current routine schedule are MMR, chicken pox (varicella), rotavirus, and the live attenuated (nasal) influenza vaccine. Rarely, other live vaccines are used for travel or in other countries. These include the BCG vaccine (for tuberculosis), the live typhoid vaccine (given orally), and the live polio vaccine (given orally.) Other kinds of vaccines, including killed virus, recombinant protein, or conjugate vaccines, are not contraindicated in immunocompromised individuals, though they may have reduced effectiveness.

A 62 year old man with a long standing history of alcohol consumption presents with dull right upper quadrant pain, jaundice and weight loss of 20 pounds over the last 6 months.Which of the following is most likely?

Liver neoplasm is often associated with cirrhosis secondary to chronic alcohol consumption; presenting symptoms may include dull RUQ pain, anorexia, weight loss, and jaundice. Diverticulitis often presents with lower left quadrant pain, changes in bowel habits (diarrhea, constipation, bleeding); Patients may have fever and an elevated white blood cell count. Small bowel obstruction often presents in those with previous abdominal surgery secondary to adhesions; Abdominal exam often reveals distention, pain on palpation (guarding, possibly rebound tenderness), and auscultation reveals hyperactive bowel sounds with high pitched tinkling, metallic sounds with air rushes. With complete obstruction or bowel fatigue, patients may have periods of absent bowel sounds. Radiographic findings on plain film may include bowel distention, "stack of coins" and multiple air fluid levels. Large bowel obstruction can present with abdominal distention, changes in bowel habits, melena or hematochezia Volvulus typically presents in older patients in their 70s, 80s, 90s, often nursing home or debilitated patients with a history of chronic constipation. They often present with progressively worsening abdominal pain (over hours), usually accompanied by abdominal distention. Abdominal plain film radiographs often reveal "coffee bean sign"; if barium enema is utilized, then characteristic "bird's beak" or "ace of spades" finding from twisted cecum or sigmoid is seen. Renal mass is often found incidentally on imaging for something else, but patients may experience flank pain, fatigue, anemia, weight loss

On physical exam, a 65 year-old female exhibits increased adipose tissue in the face, posterior neck and trunk, purple abdominal skin striae, and hirsutism. Her resting blood pressure is 162/98, fasting plasma glucose is 155 mg/dl, and she has a history of severe rheumatoid arthritis which has been treated with long-term methotrexate and prednisone. You suspect which endocrine disorder?

Long-term oral corticosteroid use can cause iatrogenic Cushing syndrome. The signs and symptoms of increased adipose tissue in the face, posterior neck and trunk, purple abdominal skin striae, and hirsutism all point to this disorder as well as the diagnoses of hypertension and elevated glucose. The other choices are not associated with these.

An 80 yo female nursing home patient s/p stroke and non ambulatory is transfered to the ED because of decreased urine output. Her mucus membranes are dry and she is tachycardic. Which laboratory finding would confirm she needs fluid replacement therapy?

Low urine sodium level indicates that the patient is dehydrated and their kidney is conserving sodium. Elevated urine sodium indicates a renal cause. The appropriate therapy is fluid replacement.

A 34-year-old female presents to your primary care office with 2 months of increasing tiredness, increased sleeping 10 - 12 hours, lack of appetite, loss of interest in her hobbies, can not concentrate on her work as an accountant, and isolation from friends and family. She moved to this location 3 months ago and missed her friends and family. Her physical examination, CBC, TSH, and CMP are all normal. Her diagnosis is:

Major Depressive Disorder (MDD) requires DEPRESSED MOOD or ANHEDONIA and FIVE or more signs andsymptoms from SIG E CAPS for a 2 WEEK PERIOD.SIG E CAPS• Sleep (Hypersomnia or Insomnia)• Interest (loss of interest or pleasure in activities = ANHEDONIA)• Guilt (feelings of worthlessness or inappropriate guilt)• Energy (decrease or fatigue)• Concentration (decrease)• Appetite (increase or decrease, weight changes)• Psychomotor agitation/retardation• Suicidal Ideations Bereavement (Simple Grief) - limited self reaction to the loss of a loved one. Patients with dysthymia present with low mood for at least 2 years as a primary symptom; they have insufficient symptoms to meet the criteria for major depressive disorder. General anxiety disorders include disorders that share features of 6 months of excessive fear and anxiety and related behavioral disturbances. These disorders include separation anxiety disorder, selective mutism, specific phobia, social anxiety disorder (social phobia), panic disorder, agoraphobia, generalized anxiety disorder, substance/medication-induced anxiety disorder, and anxiety disorder due to another medical condition.

Key clinical factors that distinguish glomerulonephritis from nephrotic syndrome include all except

Maltese cross formations in the urine are seen in nephrotic syndrome. All the other choices are seen in glomerulonephritis

A 23 year old 2 week post partum lactating female with no delivery complications presents to the ED with complaints of a very painful and swollen right breast. The patient has a temperature of 101.4 degrees F. On exam, the upper outer quadrant is indurated, red, and hot to the touch. Which of the following is the best treatment option?

Mastitis is usually caused by a staph infection and dicloxacillin is an effective antibiotic to treat mastitis. Metronidazole is used in the treatment of bacterial vaginosis and trichomonas. Erythromycin can be used as an alternative treatment if the patient is allergic to penicillin. Mastitis is an infection of the breast ducts and not a superficial skin infection. Outpatient treatment Routine cases: Amoxicillin-clavulanate 875 mg PO BID for 10-14 days Dicloxacillin 500 mg PO QID for 10-14 days or Flucloxacillin 250-500 mg PO QID for 5-7 days If penicillin intolerance (not allergy): Cephalexin 500 mg PO QID for 10-14 days If beta-lactam allergy: Clarithromycin 500 mg PO BID for 10-14 days (or see following section) If suspected community-acquired methicillin-resistant Staphylococcus aureus (CA-MRSA) infection: Clindamycin 300 mg PO TID for 10-14 days

A 35 year old female is 1 month pregnant and is in for a routine check-up. She is worried about birth defects. When is the best time to draw maternal serum alpha fetoprotein?

Maternal serum alpha-fetoprotein should be drawn around 15-18 weeks gestation. This test screens for open neural tube defects. AFP is the major serum protein in early embryonic life and is 90% of the total serum globulin in a fetus. It is believed to be involved in preventing fetal immune rejection and is first made in the yolk sac and then later in the GI system and liver of the fetus. It goes from the fetal blood stream to the fetal urinary tract, where it is excreted into the maternal amniotic fluid. The AFP can also leak into the amniotic fluid from open neural tube defects such as anencephaly and myelomeningocele, in which the fetal blood stream is in direct contact with the amniotic fluid. The first step in prenatal screening is measuring the maternal serum AFP at 15-20 weeks' gestation. A patient-specific risk is then calculated based on gestational age and AFP level.

A 54-year-old woman presents for a routine annual exam. She notes that she has been having hot flashes for several months and that her periods have become more frequent and irregular, but are lighter. Which of the following is most likely to be true regarding the patient's levels of follicle stimulating hormone (FSH), luteinizing hormone (LH), and estradiol?

Menopause is characterized by increased levels of FSH and LH and decreased estradiol levels. Laboratory markers of menopause include the following: An increase in serum follicle-stimulating hormone (FSH) and decreases in estradiol and inhibin are the major endocrine changes that occur during the transition to menopause . FSH levels are higher than luteinizing hormone (LH) levels, and both rise to even higher values than those seen in the surge during the menstrual cycle. The FSH rise precedes the LH rise; FSH is the diagnostic marker for ovarian failure, while LH is not necessary to make the diagnosis. The large cyclical variation of estradiol and estrone observed during the menstrual years ceases, and fluctuation in levels is small and inconsequential, with the mean value being considerably lower. No specific changes in thyroid function related to menopause have been found.

30 yo male diabetic presents to the ED with tachypnea and increased urinary frequency. His ABG pH 7.3, pCO2 30, pO2 100, HCO3 14 Normal values pH 7.0-----------7.4-------------7.6 PCO2 >60--------45—35----------<20 HCO3- <18--------22—26--------- >32 Your patient has:

Metabolic Acidosis with respiratory compensation - Diabetic Ketoacidosis

40 yo female presents to the ED with nausea and vomiting for 2 days. Her ABGs: pH 7.49, pCO2 47, pO2 100, HCO3 30 Normal values pH 7.0-----------7.4-------------7.6 PCO2 >60--------45—35----------<20 HCO3- <18--------22—26--------- >32 Your patient has:

Metabolic Alkalosis with respiratory compensation . Loss of HCL in vomiting has elevated her pH and bicarb. She is compensating by retaining CO2.

Your patient is a 45 year-old Caucasian diabetic male with an estimated GFR (glomerular filtration rate) of 24 mL/min in recent lab testing. Which medication is contraindicated for treatment of his type 2 diabetes?

Metformin Metformin should not be used in patients with GFR less than 30 because there is concern about increased risk of lactic acidosis in the context of renal insufficiency. Accumulation of metformin due to renal insufficiency leads to impaired hepatic metabolism of lactate, which can cause lactic acidosis.

Which of these knee concerns requires an urgent referral to an orthopedist?

Most cases of bowlegs (varus) and knock knees (valgus) in children are not pathologic and require no further intervention. Typically, newborns have some degree of varus, becoming valgus around age 3-5 years, and then straightening - but there is a lot of normal variation. Sometimes, referral is warranted for parental anxiety or if the degree of the angle interferes with a normal gait.

A 60 year old man presents with a thyroid mass for two months. Which of the following is NOT suggestive of malignancy?

Most thyroid cancers are euthyroid thus abnormal thyroid function studies are suggestive of other causes of a thyroid mass. Cancers are usually solid, not cystic; are single (as opposed to multinodular goiters); may cause hoarseness due to involvement of the recurrent laryngeal nerve; and are associated with neck irradiation (as in Chernobyl survivors)

Which is the most common side effect of the statin medication class?

Myalgia While creatine kinase is usually not elevated, many patients suffer myalgias while on statin therapy. Less commonly, CK is elevated in the case of myositis. Common myalgias (usually with normal CK) occur in 1-10% of patients on statin therapy. Changing to a different statin, lowering dose, or even considering alternate day dosing can help alleviate this adverse effect. Other effects, as listed above, have been noted less commonly or found to be unfounded in trials.

A 34 year old woman presents to the emergency department several hours after ingesting a half bottle of acetaminophen in a suicidal gesture. Which of the following is the most appropriate?

N-acetylcysteine/NAC should be given in single ingestion acetaminophen overdose secondary to life threatening hepatotoxicity, Admit patients with acetaminophen concentration above the "possible" line on the Rumack-Matthew nomogram for treatment with N -acetylcysteine (NAC). NAC is nearly 100% hepatoprotective when it is given within 8 hours after an acute acetaminophen ingestion, but can be beneficial in patients who present more than 24 hours after ingestion. NAC is approved for both oral and IV administration. The FDA-approved regimen for oral administration of NAC (Mucomyst) is as follows: Loading dose of 140 mg/kg 17 doses of 70 mg/kg given every 4 hours Total treatment duration of 72 hours The IV formulation of NAC (Acetadote) is commonly used in many institutions for the treatment of acetaminophen ingestion. Use of the IV formulation of NAC is preferred in the following situations: Altered mental status GI bleeding and/or obstruction A history of caustic ingestion Potential toxicity in a pregnant woman Inability to tolerate oral NAC because of emesis refractory to proper use of antiemetics

A 19-year-old male presents to his college student health center complaining of knee pain, swelling, and redness for the past two days. He denies any trauma or injury to his knee and states that the pain "came out of nowhere." On the intake sheet, you notice that the nurse noted that he would also like STD testing since he had unprotected sex 2 weeks ago and has noticed penile drainage. For what pathogen would you be most likely to treat given this clinical scenario? A. Staph aureus B. Neisseria gonorrhoeae C. Chlamydia D. Borrelia burgdorferi

Neisseria gonorrhoeae 1. Staphylococcus aureus: this common skin flora causes many joint infections, but without a traumatic arthrotomy or penetration of the joint and with the details of the patient's sexual history, this is less likely 2. Neisseria gonorrhoeae: gonococcal arthritis is the most common form of bacterial arthritis and is seen predominantly in young, sexually active men 3. Chlamydia trachomatis: chlamydia is commonly diagnosed and treated along with gonorrhea, but it does not cause these symptoms as commonly as gonorrhoeae 4. Borrelia burgdorferi: Lyme disease may cause monoarticular arthritis, but you would expect a recent history of travel and/or the presence of cutaneous symptoms

Which of the following is NOT associated with nephrotic syndrome?

Nephrotic syndrome does not cause hypertension. All the other characteristics are true.

A 32-year-old female patient presents to ED with agitation, fever, and muscle rigidity in both arms and legs. She does not have any headache, respiratory or GU symptoms. She is on an unknown "nerve medicine" as her only medication. She has not taken any other substances. This is a side effect of:

Neuroleptic malignant syndrome is a fatal complication of treatment with anti-psychotic medications. Various symptoms include muscular rigidity, fever, autonomic changes, agitation, and obtundation. The symptoms typically develop over 24 to 72 hours, and the untreated disorder lasts 10 to 14 days. Choice A: Antidepressants do not cause the life-threatening complication of neuroleptic malignant syndrome. Dependent upon the class of drugs to which they belong, antidepressants generally cause the following adverse effects: SSRI's: sexual side effects, decreased appetite, headache, GI disturbances, serotonin syndrome (can occur if used with MAOIs). TCAs: lethal with overdose owing to cardiac conduction arrhythmias, anticholinergic effects (dry mouth, constipation, urinary retention), confusion, weight gain MAOIs: hypertensive crisis if taken with high-tyramine foods (aged cheese, red wine, chocolate), sexual side effects Atypicals: Bupropion: decreases seizure threshold; no sexual side effects Venlafaxine, Duloxetine, Desvenlafaxine: hypertension, blurry vision, weight changes, sexual side effects, GI disturbances Mirtazapine: weight gain, sedation Nefazodone: sedation, headache, dry mouth Trazodone: highly sedating, priapism Anxiolytic side effects depend on the class to which it belongs. For instance, benzodiazepines cause sedation, confusion, memory deficits, respiratory depression, and addiction potential. Buspirone causes headaches, nausea, and dizziness A noteworthy side effect of Monoamine oxidase inhibitors (Tranylcypromine, Isocarboxazid, Phenelzine) is the interaction with tyramine-rich foods, such as cheese, red wine, and fava beans. Ingesting these foods, along with MAO-I's, can precipitate a hypertensive crisis. A hypertensive crisis is a life-threatening emergency. Other unfavorable side effects include: weight gain, insomnia, decreased libido, and orthostatic hypotension.

Your patient is a 32-year-old male who is regularly washing his hands to the point that he has bilateral hand dermatitis. He feels a compulsion to wash his hands every few minutes. What class of drugs is considered to be first-line drug treatment?

Obsessive-compulsive disorder is a disorder in which patients experience either obsessions alone or, most commonly, a combination of obsessions and compulsions. SSRIs are the treatment of choice. Fluoxetine, Paroxetine, Sertraline, Citalopram, or Fluvoxamine are most commonly used as first-line agents. Atypical antidepressants, such as Bupropion, Mirtazapine, and Trazodone, are not indicated to treat obsessive-compulsive disorder. Bupropion is often the drug of choice for patients who are depressed, overweight, and want to quit smoking. Mood stabilizers, such as Lithium, Valproic Acid, and Lamotrigine, serve no purpose in treating obsessive-compulsive disorder. These drugs are used more commonly in bipolar disorder. MAO-Is are used to treat depression when other drugs, such as SSRIs, have not been effective. A special diet is needed when using MAO-I's

Which of the following pediatric rashes is typically itchy?

Of these choices, only eczema is typically itchy. Other characteristically itchy rashes of childhood include hives (urticaria), contact dermatitis (including poison ivy), and the rash that accompanies scabies infestations.

A 55 year old man with atrial fibrillation presents with complaint of acute visual changes within the past day, "like looking through a glass of water." He denies eye trauma or headache. On exam his visual acuity is normal, and pupils are 3mm and reactive to light. Extraocular movements are intact. However on visual fields testing by direct confrontation he has a field deficit of the left visual field of each eye. Based on these findings, where is the problem in the visual fields pathway?

Optic radiation or occipital cortex A homonymous hemianopsia is caused by lesion at the optic radiation or occipital cortex. This causes a defect in the contralateral visual field in both eyes. That is, a lesion in the right occipital lobe will cause a deficit of the left visual field. Conveniently, the deficit is analogous to that of motor weakness; a stroke on the right side of the brain will cause weakness in the left side of the body. The lesion of the retina or optic nerve will cause monocular visual problems. A lesion at the optic chiasm will cause bitemporal visual loss.

A 29-year-old female presents with h/o sudden and severe pain in her lower abdomen. She has experienced nausea and vomiting with the pain. The patient has a history of Polycystic ovary syndrome or PCOS. You have a high suspicion of ovarian torsion. The best method to diagnose an ovarian torsion is

Ovarian torsion occurs when an ovary becomes twisted around its pedicle and the blood supply is cut off to the ovary. The best method to diagnose an ovarian torsion is an ultrasound with color doppler because it will show a lack of blood flow to the ovary. Laparoscopic surgery or a laparotomy are the surgical treatment options after a diagnosis has been made.

What is the classic abortive therapy for cluster headaches?

Oxygen The cluster headache is unique in its response to 100% oxygen high flow by facemask. NSAIDs are the mainstay of treatment for tension headaches and may be effective for other headaches as well. Ergotamine is an older migraine abortive. Sumatriptan is the first line abortive treatment for migraine headaches.

A 20-year-old female patient presents to the ED with increasing fatigue and bruising with minimal injury. She is having several nose bleeds. A CBC reveals low RBCs, WBCs, and platelets. What diagnostic step is most appropriate?

Pancytopenia, where all three cell lines are low, is an indicator of bone marrow failure. The causes could be: Myelodysplastic syndromes, leukemias, immune - drug-mediated, or toxins The best diagnostic test is a bone marrow biopsy and pathologist review

A 21-year-old female college student presents to the ED with a 5-minute episode this morning of non-exertional palpitations, chest tightness, overwhelming anxiety, and sweating. She had four similar episodes occurring at home with no precipitating events over the last month. Her ROS, PMH, and Social Hx are noncontributory. She does not take medications or use drugs, caffeine, or alcohol Her VS: pulse 90 regular Resp 16, BP 120/68 Pulse Ox 98% PE: all normal Labs CBC, CMP, d-dimer all normal Her diagnosis is:

Panic Attacks - 3 attacks within 3 weeks lasting at least 10 minutes or patient presentswith 1 or more months of concern about having additional attacks. *Extreme anxiety*Feelings of impending doom*Chest pain*Palpitations*Diaphoresis*Nausea/Dizziness*Feeling of losing control*Chills or hot flashes Short-term therapy:-Benzodiazepines (Clonazepam) only for immediate release-Taper Benzos as soon as long-term tx(SSRIs) started*Long-term therapy: -CBT-Medications (SSRIs first-line, TCAs)

A 48 year old causasian male has a recent PSA elevation to 5.6 ng/mL. You perform a digital rectal examination and find a hard nodule to his left peripheral zone. His father has a history of prostate cancer at age 50. What is the next best step. The patient does take a oral DOAC anticogulant for his past history of pulmonary embolus. PSA levels chart Age Range (Years)Asian Americans Caucasians 40 to 490 to 2.0 ng/mL0 to 2.5 ng/mL50 to 590 to 3.0 ng/mL0 to 3.5 ng/mL60 to 690 to 4.0 ng/mL0 to 4.5 ng/mL70 to 790 to 5.0 ng/mL0 to 6.5 ng/mL

Patient has concerning PSA and lesion palpated on prostate exam with significant familial history of prostate cancer. He is high risk for a clinically significant prostate cancer. Because of this, you should obtain a prostate biopsy immediately. Waiting for MRI could delay his care. Would need to hold his DOAC for procedure. Repeating a PSA would not help as you felt a nodule. Prostatectomy possible in future but extent of prostate cancer needs to be determined first.

68 year old male with BPH presents to the ED in urinary retention. The ED staff is unable to place a catheter so they call you to assist. Which catheter should you consider placing.

Patient has obstructive BPH. A male coude catheter has a curve to the tip of the catheter that allows placement around the prostate. While a silicone catheter may be helpful, it does not have the curvature that the coude does. 3-way catheter is not needed and 12 French catheter is too small and not firm enough to place.

A 28 year old female presents to ED with fever of 101.1 F with left flank pain and nausea/vomiting. CT scan reveals an obstructing 4mm left ureterovesical junction stone with upstream hydronephrosis and stranding around the kidney. What is the best management?

Patient should not be discharged as she has Pyelonephritis. Important to note that Diagnosis of Pyelonephritis is clinical, not based just on imaging. Fever with UTI and flank pain is enough to diagnose pyelonephritis. She does need IV antibiotics. While the kidney stone may be the culprit, performing a Ureteroscopy with laser lithotripsy in the setting of infection may cause patient to become septic.

A 22 year old man presents to the emergency department with a painful, warm, red fluctuant mass superior to his intergluteal cleft. Which of the following is the most likely diagnosis?

Pilonidal cyst is a painful, swollen lesion in the intergluteal area, sacrococcygeal region about 4-5 cm posterior to the anal orifice. At times, spontaneous drainage may have occurred prior to presentation to the clinician. Patients may present with intermittent swelling and drainage, including purulent, mucoid, or bloody fluid from the area. Chronic pilonidal disease often manifests as recurrent or persistent drainage and pain. Hidradenitis suppurativa does have abscess formation but is usually more multiple rather than singular and affects the axilla, inner thighs, inguinal, and perianal area. Perianal abscess does have abscess formation but affects the perianal area. External hemorrhoids are located below the dentate line and are covered by anal skin, they are usually itchy and burning. Perirectal abscess is a more severe problem as it surrounds the rectum and is more internal, often requiring admission, antibiotics and operative drainage.

Which medication is contraindicated in a patient with type 2 diabetes and stable Class 3 congestive heart failure with no hospitalizations within the last year?

Pioglitazone (a thiazolidinedione) is contraindicated in Class 3 CHF because it can worsen edema and fluid accumulation, thus leading to worsening heart failure. In Class 1 and 2 CHF, TZDs should be used with caution, but may be considered at lower dosing. In addition, metformin should be used with caution in patients with CHF, but is not contraindicated in stable patients.

62 year-old female patient presents with morning stiffness of more than one hour duration with symmetrical swelling of wrists, MCPs, and PIPs, but sparing of DIPs. She complains more about her limited ADLs than pain. This patient likely suffers from

Rheumatoid arthritis is correct. Signs and symptoms of RA may include the following: Persistent symmetric polyarthritis (synovitis) of hands and feet (hallmark feature) Progressive articular deterioration Extra-articular involvement Difficulty performing activities of daily living (ADLs) Constitutional symptoms Osteoarthitis commonly effects the DIP joints of the hand. Septic arthritis is usually monoarticular and painful. Gout is usually monoarticular, painful and not very common in women (but not unheard of either). Pseudogout is common in both genders, especially in the elderly, but is painful and usually effects large joints (i.e. knee).

The most common cause of secondary amenorrhea is

Pregnancy is the most common cause of secondary amenorrhea. Polycystic ovary syndrome or PCOS and anorexia can also be causes of secondary amenorrhea, but not the most common cause. Endometriosis tends to create longer cycles.

Which set of risk factors requires the lowest (or most stringent) LDL, non-HDL, and Apo-B goals?

Progressive or premature ASCVD All are risk factors, but the most extreme risk category is those with progressive or premature ASCVD. All of these risk factors require treatment.

NSAIDs block prostaglandin production to reduce pain and inflammation. Which of the following is not true regarding the physiologic "good housekeeping" effects of prostaglandins?

Prostaglandins exhibit GI protective effects by stimulating HCL secretion is not true, All the other choices are true regarding the physiologic "housekeeping" effects of prostaglandins. Prostaglandins function as autacoids responsible for normal physiologic functions in the gastrointestinal tract, kidney, reproductive system, platelets, and vasculature. Prostaglandins (and thromboxane) are also key mediators of inflammation and pain

Which of the following is not true of gynecomastia?

Pubertal gynecomastia usually appears when a boy is at Sexual Maturity Rating (SMR) 2 or 3. It may be unilateral or bilateral. It can be caused by medicines like neuroleptics that increase serum prolactin, but usually no specific cause is identified, and no treatment other than reassurance is needed.

A baby has projectile vomiting. Which of the following suggests a diagnosis of pyloric stenosis?

Pyloric stenosis is an acquired gastric outlet obstruction. Symptoms usually begin a week or more after birth. Since the obstruction is at the pylorus, proximal to the bile duct, the vomitus does not contain bile. Babies with PS feed ravenously, and do not gain weight well. Classically, blood tests will reveal a hypochloremic metabolic alkalosis (with a low serum chloride, and a high serum bicarbonate.)

Which of the following is NOT an objective symptom of pregnancy?

Quickening- sensation of fetal movement- is a subjective symptom of pregnancy. Hegar's sign, softening of the isthmus of the uterus, and Chadwick's sign, bluish discoloration of the cervix, vagina, and labia, are objective symptoms of pregnancy as is a darkened breast areola region. Goodell's sign, softening of the cervix, is also an objective symptom of pregnancy.

A 5-year-old boy has a history of recurrent knee hemarthroses and excessive nosebleeds. The clotting time/PFA is normal and the PT is normal. The aPTT is prolonged but corrects with a mixing study. Which of the following tests will be most revealing of his diagnosis?

Recurrent hemarthrosis indicates a factor deficiency. The most common genetic factor deficiency is chromosome X linked recessive factor VIII or hemophilia A, followed by factor IX hemophilia B. These two factors are in the intrinsic pathway, best detected by a prolonged aPTT. Factor X deficiency is rare. The TT measures factors in the common pathway; the normal PT indicates a normally functioning common pathway and factor VII is working.

A 63 year old female arrives with acute onset of right facial weakness, difficulty producing speech and has profound weakness of the right arm. She has not been taking her blood pressure medications. Imaging shows an ischemic stroke in the territory of the left middle cerebral artery. Blood pressure is 200/110. What is the first treatment goal?

Reduction in blood pressure to 170/90 This patient has uncontrolled hypertension and now has a stroke, which is a form of brain injury. She may have increased intracranial pressure, and she may need "permissive hypertension" to perfuse her injured brain. Be careful not to "overcorrect" the hypertensive patient with a stroke, as you could actually worsened the ischemic insult to the brain. After she is recovered, she will benefit from interventions to make her more normotensive. Anticoagulation is not a standard treatment for ischemic stroke (unless the stroke is caused by atrial fibrillation, in which case the anticoagulation therapy can be started non-emergently). Cerebrovascular goals for glucose and LDL are similar to cardiovascular goals.

An 18 yo male has newly discovered hypertension. His physical exam is normal except you hear a bruit in the upper left quadrant of his abdomen. What diagnosis should you investigate next

Renal artery stenosis should be considered in a patient with hypertension and an abdominal bruit. Renal artery stenosis (RAS) is the major cause of renovascular hypertension and may account for 1-10% of the 50 million cases of hypertension in the United States population. Acute kidney injury or decreased renal function can occur after initiation of antihypertensive therapy, especially with angiotensin-converting enzyme (ACE) inhibitors or angiotensin receptor blockers. These should be avoided until RAS is ruled-out.

A 30-year-old female is being presents a one-year history of her fingers turning white then red with pain in cold weather. She also has arthritis involving PIP joints, wrists, ankles, and knees. On physical examination sclerodactyly, ulceration of fingertips, and swelling and tenderness over PIP joints were noted. What is the most likely diagnosis?

Scleroderma Dx: 1 major or 2 minor criteria •Major -thick, tight, induration of skin of fingers and skin proximal to the MCP or MTP joints; may affect entire extremity, face, neck, and trunk •Minor -Sclerodactyly= thickening, induration, and tightening of skin, limited to fingers. -Digital pitting scars or loss of substance from the finger pad due to ischemia -Bibasilar pulmonary fibrosis includes bilateral reticular pattern of linear or lineonodular densities (honeycomb lung) not attributable to primary lung ds -Pigmentation changes, itching, calcinosis cutis -Raynaud phenomenon, telangiectasia, dry gangrene -GERD, dysphagia, malabsorption, Primary Biliary Cirrhosis -Restrictive lung disease -Arthralgia, myalgia, ↓ROM, Carpal Tunnel Syndrome, weakness, sclerodactyly -Hypothyroidism -HTN, renal insufficiency/crisis -Cor pulmonale(RHF), cardiomyopathy -Fatigue, weight loss -Dry eyes, mouth CREST syndrome (calcinosis, Raynaud phenomenon, esophageal dysmotility, sclerodactyly, and telangiectasias—although not all are needed for the disorder to be called CREST)

24 year old male presents with soft, mobile, somewhat tender testicular mass that has been present for the past 3 weeks. Denies any recent sexual encounters, no history of mass in the scrotum/testicles. Is an avid runner. What is the most likely diagnosis?

Soft, mobile, tender lesions are usually not testicular cancer. In this case obtaining an U/S could help diagnose an epididymal cyst but this diagnosis can be made clinically as well. He shows no signs of infection and has no recent sexual encounters so epididymitis is unlikely. Torsion presents as severe pain, usually unilaterally without relief with elevation of the testicle.

Which stage of normal labor begins with complete dilation and effacement of the cervix and ends with the delivery of the baby?

Stage 1 begins with regular contractions and ends with full dilation of the cervix. Stage 2 begins with complete dilation and effacement of the cervix and ends with the delivery of the baby. Stage 3 begins after the delivery of the baby and ends with the delivery of the placenta. Stage 4 is the first hour after delivery to assess bleeding and treat any lacerations, tears, or episiotomies.

A 65 year old female presents the ED with bilateral staghorn calculi. She continues to have recurrent UTIs and would like something done. What options would be available to her?

Staghorn calculi are only amendable to Staged Per Cutaneous Nephrolithostomy PCNL. Ureteroscopy with Laser Lithotripsy would not provide enough access to successfully breakup the stone. Electro Shockwave Lithotripsy is not indicated for staghorn calculi. Medical Expulsive Therapy would not work to pass the stones.

A 2 year old boy is noted to have crossed eyes on his examination. Which answer(s) is(are) true about strabismus and amblyopia? 1. Patching can correct strabismus 2. Patching can correct amblyopia 3. Glasses can correct some causes of strabismus 4. Glasses can correct some causes of amblyopia

Strabismus, or crossed eyes, is sometimes caused by a refractive error, and in this case it can be corrected with glasses. If untreated, strabismus can lead to amblyopia (lazy eye). Another potential cause of amblyopia is a high refractive error causing bilateral or unilateral blurred vision. Patching of the "good eye" can help correct some kinds of amblyopia, as can improving visual sharpness with glasses. Patching, itself, does not correct strabismus.

An 18 year old man is brought to the emergency department by emergency medical services after sustaining a penetrating gun shot wound to the abdomen. His vitals signs are BP 70/30 mmHg, heart rate 150/min, respirations 28/min. Which of the following is most appropriate for this patient at this time?

Tachycardia, tachypnea and hypotension are considered unstable vital signs in penetrating abdominal trauma and unstable patients should NEVER be sent for CT but instead go directly to the operating room Immediate surgical exploration is warranted for evidence of significant intra-abdominal injury, especially vascular trauma, such as the following: Hypotension (with or without abdominal distention) Narrow pulse pressure Tachycardia High or low respiratory rate Signs of inadequate end organ perfusion Peritoneal signs (eg, pain, guarding, rebound tenderness) and/or peritonitis Diffuse and poorly localized pain that fails to resolve

A 25-year-old presents to the ED with a 1-day history of involuntary facial movements that he can not stop. He is taking unknown prescription medication for his "Nerves" for several months from his psychiatrist. Which of the following category of drugs is known to include this side effect?

Tardive dyskinesia is a late-onset development of involuntary, choreoathetoid movements, particularly of (but not limited to) the face, orobuccal region, neck, fingers, and toes. It is usually irreversible. Risk factors include long-term treatment with neuroleptics. Typical antipsychotics are notorious for causing this disorder. The longer a patient is taking an antipsychotic, the more risk is associated with developing tardive dyskinesia. The degree of these movements varies from negligible to grossly severe. If the disorder is severe enough, action should be taken to stop the particular antipsychotic and replace it with one of the atypical antipsychotics. Typical neuroleptics carry a 5% per year risk for tardive dyskinesia and atypical neuroleptics, much less, usually considered 0.5% per year or lower. Side effects of benzodiazepines include sedation, confusion, memory deficits, respiratory depression, and addiction potential.MAO-Inhibitors cause orthostatic hypotension, weight gain, edema, sexual dysfunction, and insomnia. Patients should be advised that caution should be taken with food rich in tyramine, such as aged cheese, red wine, and chocolate. When these sorts of food are eaten along with taking an MAO-I, a hypertensive crisis can ensue.Tricyclic Antidepressants cause hypotension, dry mouth, constipation, confusion, arrhythmias, sexual side effects, weight gain, and GI disturbances.

A 18-year-old healthy male has been mildly anemic all his life. Today his hemoglobin is 11 g/dL (13.5 to 18 g/dL) and his MCV is low at 72 (80 to 100 fL); ferritin is normal at 35 (20 to 250 ng/mL). Which of the following is the most appropriate diagnostic test?

Thalassemia is the most common hereditary anemia, causing a microcytic cell size and a normal ferritin. Normal ferritin indicates that this is not iron deficiency. A peripheral smear would confirm the microcytic anemia but lend no additional information. The differential diagnosis of microcytic anemia is remembered by the mnemonic "TICS" T - Thalassemia - Dx hemoglobin electrophoresis I - Iron Deficiency - Dx Iron studies and low ferritin C - Chronic inflammation - Dx Elevated c-reactive protein S - Sideroblastic - lead toxicity is the most common cause - Dx lead level A hemoglobin electrophoresis would be the next step to diagnose beta thalassemia. Heinz body stain-positive G6PD deficiency causes a hemolytic anemia when the patient is exposed to oxidant medications, like sulfa or antimalarials, or eating Fava beans. Folate deficiency causes a macrocytic anemia. Erythropoietin would be decreased in chronic renal failure resulting in a normocytic anemia.

Which of the following children needs an urgent medical evaluation including an MRI of the pituitary?

The 8 year old boy with large testes and phallus has clearly started puberty. That is early, and boys with early puberty need a thorough evaluation including imaging. A boy or girl who develops only scant pubic hair at an early age is unlikely to be experiencing true puberty, and watchful waiting is appropriate. A 14 year old boy who has not yet started puberty is on the late side, but this is not an urgent situation.

A 15-year-old male presents with discoloration on his eyelids and hands and trunk over the past year. He denies pain or itch and has not used any over the counter or prescription treatments. Upon examination of the skin, depigmented macules and patches are noted over bilateral lower eyelids, lips and dorsal aspects of fingers and hands. The affected areas on his hands exhibit depigmented hairs. This patient's suspected diagnosis puts him at risk for which of the following?

The correct answer is Autoimmune thyroid disease. Patients with one autoimmune condition, such as vitiligo, have up to a 25% risk of being affected by other autoimmune disorders such as thyroid disease, type 1 diabetes, rheumatoid arthritis, lupus, Addison's disease, and pernicious anemia. Incorrect answers: Type 2 diabetes mellitus is not associated; however, vitiligo patients are at an increased risk for type 1 diabetes mellitus. Leprosy is an infectious process that can cause light patches of skin. By contrast, vitiligo is an autoimmune disease not associated with Leprosy. Albinism is brought on by a genetic mutation causing little to no production of melanin pigment in the skin, hair and eyes which is present at birth.

Which of the following conditions is likely to present with both nail dystrophy and white lacy patches of the lips, tongue, or oral mucosa?

The correct answer is Lichen Planus which often presents with milky white lacy patches of the buccal mucosa, lips and/or tongue. It is also associated with nail changes including ridging, thinning, cuticle destruction and pterygium formation. The skin lesions of lichen planus appear as plentiful purple, planar, polygonal papules. Incorrect answers: Psoriasis often presents with nail changes such as nail pitting, ridging and dystrophy, however lesions of the oral lesions are not characteristic of this disease process. Onychomycosis causes yellowing, dystrophy and thickening of the nails but oral lesions are not present. Pemphigus vulgaris may cause ulceration of the oral mucosa but nail changes are not commonly associated.

A 4-year old, non-vaccinated boy presents to the clinic with a 4-day history of fever, sore throat, dry cough, and runny nose. Yesterday, his mom noticed he developed a rash on his face that has now spread to his trunk and extremities. His temperature is 103 degrees Fahrenheit. Upon physical examination, he has a non-purulent conjunctivitis as well as erythematous, confluent macules and papules of the face, trunk and extremities, including feet. Examination of the buccal mucosa reveal, pinpoint whitish-grey speckling with erythematous halos. Given the physical exam findings, which of the following is the most likely diagnosis?

The correct answer is Measles which initially presents with cough, coryza, and conjunctivitis in an unvaccinated individual. Within 3-5 days of symptoms, an exanthem of macules and papules develops over the hairline and post-auricular region and spreads to the trunk and extremities distally. Koplik spots of the buccal mucosa are highly characteristic of measles. Incorrect answers: Erythema infectiosum presents with fever, slapped red cheeks and a lacy rash of the trunk and extremities. Roseola infantum presents with high fever, sore throat, cough, and congestion. As the fever resolves, patients develop an erythematous, macular rash that mainly involves the trunk. Rubella presents with mild symptoms of low-grade fever, headache, sore throat, and runny nose. Pinpoint petechiae develop over the soft palate and uvula (Forchheimer's spots). Exanthematous fine macules and papules develop cephalocaudally.

A 30-year-old pregnant woman presents to your office with a 3-month history of skin discoloration on her face. She has tried using exfoliating cleansers and sunscreen but states that the condition has not improved. Upon examination of the skin, well demarcated, evenly pigmented, tannish-brown patches without scale are noted over the forehead and upper cutaneous lip. Which of the following is the most likely diagnosis?

The correct answer is Melasma, a brown patchy discoloration of the face and forearms, also known as the "mask of pregnancy". Melasma is caused by an increase in hormones in pregnant women. Non-pregnant women also experience melasma, especially in those taking oral contraceptive pills or hormone replacement therapy. Sun exposure also plays a large role in the development of melasma. Incorrect answers: Acanthosis nigricans is commonly noted within the skin folds of the neck and extremities which present as hyperpigmented, velvety patches and possible skin tag formation. Vitiligo presents as sharply demarcated patches of depigmentation, rather than hyperpigmentation. It presents in a "lip-tip" symmetric pattern of the eyes, mouth, hands, elbows, knees, nipples, and genitalia. Malignant Melanoma is a malignancy of the skin which presents as an ill-defined, asymmetric macule or papule with uneven pigmentation.

A 10-year-old girl presents to the clinic with a 4-month history of a bumpy rash that is spreading. The lesions are asymptomatic but bothersome in appearance. Mom states she noticed the lesions since her daughter started taking gymnastics classes.Upon examination of the skin reveals multiple, pink, umbilicated papules over the abdomen, thighs, and axillary region.Which of the following is the most likely diagnosis?

The correct answer is Molluscum Contagiosum which is part of the pox virus family. These highly contagious, umbilicated papules spread by direct contact among young children and in sexually active adults. Incorrect answers: Verruca Vulgaris is caused by the human papillomavirus, causing firm, hyperkeratotic papules commonly noted on the hands and feet as well as other sites of the body. Varicella Zoster, also known as chicken pox, is spread by airborne droplets and secretions resulting in a vesicular eruption, described as "dew drops on a rose petal".It initially presents on the face, then spreads to the trunk and extremities. Herpes Simplex is a recurring viral infection characterized by painful, grouped vesicles on an erythematous base commonly involving the face and mouth in children.

A 32-year-old woman presents for evaluation of an itchy rash that has been present for more than 5 years. The patient states the rash started on her scalp and has spread to involve her arms and legs. She has applied various over-the-counter remedies to this rash with no improvement of her symptoms. The patient states her sister has a similar rash which has been present for years.This patient denies fever, recent illness, seasonal allergies, asthma, joint pain, or muscle aches. Upon examination of the skin, well demarcated, thick, silvery, scaling plaques are noted throughout the scalp, lumbosacral region, bilateral elbows, and knees. She has pitting dystrophy of the fingernails.Examination of the oral mucosa is within normal limits. Given these findings, which of following is the most likely diagnosis?

The correct answer is Plaque psoriasis. Psoriasis is an inherited disorder characterized by pruritis, inflammation and hyperproliferation of skin that occurs commonly in sites of trauma and presents with symmetric, erythematous, silvery, scaling plaques with nail pitting and dystrophy. Incorrect Answers: Lichen Planus in an inflammatory eruption of the skin that often involves the mucous membranes (Wickham's striae) and presents with flat topped, violaceous papules and plaques over the wrists and forearms. Nail changes include ridging, thinning or complete destruction/loss. Nail pitting is not present, and it is not seen in other family members. Atopic Dermatitis presents with pruritic, poorly demarcated patches of erythema and scale classically noted over antecubital fossae, popliteal fossae, face, neck, and hands in those with a history of atopy (seasonal allergies, asthma, and/or sinusitis). Lichen simplex chronicus is a well demarcated localized annular plaque of lichenification occurring in 1 or more areas of habitual rubbing and scratching without the propensity for familial inheritance.

A 19-year-old boy, who is home from college for the summer, presents with an itchy rash "all over" for 1 month. He states he his roommates and girlfriend also have an itchy rash.He believes it may be related to using a new laundry detergent. Upon examination of the skin, multiple small excoriated papules and burrows are noted over the finger web spaces, wrists, lower legs, and scrotum. What is the most likely cause of this patient's rash?

The correct answer is Sarcoptes scabiei, a contagious, parasitic mite that burrows into the skin causing scabies. Incorrect answers: Phthirus pubis is the louse associated with pediculosis pubis/pubic lice affecting the hair bearing areas of the body. Cimex lectularius, also known as bed bugs, presents as inflamed papules noted in a linear array on the trunk and/or extremities. Pediculus humanus capitis is the louse associated with pediculosis capitis/head lice and is noted in the hair bearing areas of the scalp and/or eyelashes.

A 27-year-old man presents with a lesion of the left lateral buttock that has been present for years. The patient states this lesion has been slowly growing in overall size but denies associated pain or swelling. Upon examination of this lesion, a 2 cm non-inflamed, symmetric, subcutaneous nodule with a central pore is noted over the left lateral buttock.Which of the following, is the most likely diagnosis?

The correct answer is Sebaceous cysts, which are common, benign, cutaneous cysts that form by invagination of the dermal epithelium connected to the surface of the skin by a central pore. Incorrect answers: Lipomas are common, benign, subcutaneous tumors of fat. A prominent, central, overlying pore is not characteristic of this lesion. A pilonidal cyst is an anogenital cyst located at the superior aspect of the gluteal cleft near the coccyx. These lesions are often inflamed and may present with hair projecting from the mass. Squamous Cell Carcinoma presents as an indurated scaling papule, plaque or nodule commonly noted on sun exposed areas of the scalp, head, neck, or extremities.

A 66-year-old woman presents with an extremely itchy rash of the trunk and extremities which started 2 weeks ago. She states this rash comes and goes daily and has affected her ability to sleep at night.The patient states the rash began after starting an oral antibiotic prescribed by her dentist for a tooth abscess.Upon examination of the skin, the patient has no identifiable rash over the trunk or extremities. The patient demonstrates that scratching the skin on her arms, makes the rash return. What is the most diagnosis for this patient's rash?

The correct answer is Urticaria which presents with transient wheals that may not be present during physical examination of the skin. Dermatographism, or stroking of the skin, can elicit hive formation. Incorrect answers: Pityriasis rosea is a self-limited eruption of erythematous annular plaques with collarettes of scale. It is typically present for 6-8 weeks before resolving. Stevens-Johnson syndrome is a life-threatening reaction, most commonly caused by a medication. SJS quickly progresses from confluent target lesions to flaccid bullae which spread upon lateral pressure with sheet-like necrosis of the skin. Erythema multiforme minor forms target shaped macules, papules and plaques that do not tend to wax and wane during a 24-hour period.

Your patient has the following rash on one side of his neck and face. Which of the following medications is the most appropriate initial therapy for this patient's current condition? [Picture has red patches on right side of face]

The correct answer is Valacyclovir an anti-viral medication commonly used in the initial management of Herpes Zoster. Anti-viral therapy aids in shortening the course of the disease process, healing lesions faster and providing relief from pain. Incorrect Answers: Prednisone is a corticosteroid that may aid in reducing pain associated with herpes zoster neuritis but is not considered a first line treatment. The use of prednisone in herpes zoster is controversial and is only used in combination with antiviral therapy. Cephalexin is an oral antibiotic that is not indicated for the treatment of viral infections such as herpes zoster. Terbinafine is an anti-fungal therapy that is not indicated for the treatment of viral infections such as herpes zoster.

Which of the following conditions is characterized by annular, poorly demarcated patches of scale noted in the flexures of the arms and legs in a patient with a history of sensitive skin, allergic rhinitis, and asthma?

The correct answer is atopic dermatitis, a common rash characterized by poorly demarcated scaling patches in those with a history of sensitive skin, asthma, and/or seasonal allergies. The flexures of the skin are commonly affected. Incorrect answers: Seborrheic dermatitis (dandruff) presents with fine scale, erythema and greasy patches over the scalp, eyebrows, nasolabial folds, postauricular folds and central chest in patients with increased sebum production and/or poor hygiene. Tinea corporis is a fungal infection of the skin, presenting as well demarcated annular plaque(s) with or without central clearing. Scabies is a parasitic infection characterized by intensely pruritic papules, burrows, and excoriations. It is commonly seen over the web spaces of the hands, trunk, and extremities.

A 47-year-old man comes to the office with a 2-day history of a painful rash on his trunk. The patient is afebrile. The patient states he is experiencing a burning sensation and can hardly tolerate clothing resting against his skin. The patient admits to a lot of stress recently related to his job. Use of over-the-counter ibuprofen has not provided relief. On physical examination of the skin, erythematous, grouped vesicles and crusts are noted on the right lower abdomen, right flank, and right lower back. Which of the following diagnostic tests is the most appropriate initial test for this patient's condition?

The correct answer is option 3: Tzanck Smear. This question assesses the ability to recognize herpes zoster as the cause of this patient's symptoms and physical examination findings. A tzanck smear is a rapid, inexpensive, diagnostic tool that uses giemsa stain to detect the microscopic presence of multinucleated giant cells, which is diagnostic for herpes virus infections. Incorrect Answers. A KOH prep test utilizes potassium hydroxide solution microscopically to diagnose superficial fungal infections of the skin. An aerobic bacterial culture is appropriate for isolating and identifying aerobic bacterial pathogens causing infections of the skin and does not diagnose viral infections. A punch biopsy is not appropriate as an initial test because it is a full thickness sampling of the skin. This procedure is a more costly procedure that may cause scarring and delay in diagnosis.

Which of the following conditions is most likely to occur in an adult presenting with pruritus, erythema, and fine scale on the scalp, postauricular folds and t-zone of the face?

The correct answer is seborrheic dermatitis which starts in puberty and continues into adulthood. It is characterized by fine scale and erythema of the scalp, eyebrows, nasolabial folds, postauricular folds and central chest where the sebaceous glands predominate. Incorrect answers: Psoriasis presents as thickened plaques of erythema and silvery scale commonly noted on the scalp and has a symmetric distribution over trunk and extremities. Tinea Capitis is more common in children than adults and typically presents with erythema, scale and hair loss of the scalp and cervical lymphadenopathy. Pityriasis Rosea initially presents with a salmon colored herald patch on the trunk or extremities which quickly evolves into a widespread eruption of plaques with collarettes of scale. The face and scalp are typically spared in pityriasis rosea.

A 47 year old woman presents to the emergency department with complaints of right upper quadrant abdominal pain worse after a fatty meal. She has experienced similar episodes in the past but they typically resolve within about two hours. This time the pain hasn't subsided and she has noted several episodes of vomiting and feeling feverish. Which of the following physical examination techniques may help confirm her diagnosis?

The correct answer: Murphy's sign is associated with acute cholecystitis which is classically described in the question. Rovsing's sign is pain in lower right quadrant when lower left quadrant is palpated and may be associated with appendicitis. Shifting dullness is associated with liver problems and ascites. Psoas sign is positive when a patient reports tenderness when the psoas muscle is contracted or stretched from peritoneal irritation from an inflammed appendix lying over it, associated with appendicitis. Obturator sign is positive when a patient reports tenderness when the obturator internus muscle is contracted or stretched from peritoneal irritation from an inflamed appendix lying over it, associated with appendicitis.

Which analgesic is most appropriate for a patient with elevated BUN/Creat level?

The correct choice is acetaminophen. All of the NSAIDs can decrease renal perfusion and increase kidney injury.

A 22-year-old female presents to the clinic with a rash that developed 3 days after getting a tattoo on her right scapula. She states the rash is mildly painful and spreading. Upon examination of the skin, she is found to have honey-colored crusts and erosions over the tattoo site as well as the upper back, right superior shoulder, and right upper arm. Which of the following is the most likely diagnosis?

The correct diagnosis is Impetigo, a superficial infection of the epidermis commonly caused by Staph aureus, which presents with honey colored crusts and erosions. Incorrect answers: Allergic contact dermatitis is a delayed hypersensitivity reaction to an allergen that causes itching as well as erythema, scale, and/or bullous lesions at sites of direct exposure. Cellulitis is an acute bacterial infection involving the deeper subcutaneous layers of the skin, causing a red, hot, tender, swollen area over one extremity. Bullous Pemphigoid is a chronic, autoimmune, bullous disorder often seen in the elderly and is not a complication of new tattoos.

A 35 y.o. female presents with a 1 year history of facial redness, flushing & blushing which appears to be triggered after eating certain foods and exercise. She also complains of acne on her cheeks & nose. On examination of the face, the patient has erythema, papules & pustules noted predominantly over the nose & cheeks bilaterally. The most likely diagnosis is:

The correct diagnosis is rosacea. Rosacea is a common condition characterized by symptoms of facial flushing and a spectrum of clinical signs, including erythema, telangiectasia, coarseness of skin, and an inflammatory papulopustular eruption resembling acne. Acne presents with comedones, papules or pustules over the t-zone & cheeks without flushing & blushing. SLE is an autoimmune condition with the potential for multi-organ involvement & presents with a pruritic erythematous butterfly rash of the mid face in the absence of acneiform lesions. Allergic Contact Dermatitis is an inflammatory skin response to an allergen causing pruritic, edematous, erythematous papules &/or vesicles. Eczema is characterized by poorly demarcated patches of dry scaling papules & patches commonly seen in those with allergies, asthma, sinusitis, etc.

A 55 yo female with hypertension taking hydrochlothiazide presents to the clinic with 1 week of weakness and fatigue with doing her regular chores. She is also reporting constipation and abdominal cramps. Her EKG shows prominent U waves, flat T waves, and ST depression on EKG may suggest which of the following electrolyte abnormalities?

The earliest EKG pattern seen with hypokalemia (low potassium) is a decrease in the amplitude of the T wave. As the potassium level declines the ST segment becomes depressed with eventual T wave inversion.The PR segment may become prolonged.The U wave is a positive deflection that follows the T wave (may initially take on the appearance of a humped wide T wave), often best seen in the mid pre-cordial leads. When the size of the U wave exceeds the T wave, the potassium level is < 3 mEq/L. The EKG pattern seen in Hyperkalemia is typically depicted by tall peaked T waves seen diffusely, which progress to a shortened QT interval, lengthening of the PR interval with loss of the P waves, and then widening of the QRS complex.. Hypocalcemia is manifested by lengthening of the ST segment and eventually in a prolonged QT interval. Hypomagnesaemia primarily cause arrhythmias.

Metronidazole 2 g orally as a single dose or 500 mg twice daily for 7 days is the treatment regimen for which of the following vaginal infections?

The high-dose treatment by a single 2 g oral dose of metronidazole is used for the treatment of Trichomoniasis. An alternative multi dose regimen is metronidazole 500 mg orally twice a day for seven days. Trichomonas is a sexually transmitted infection. Clinical characteristics include a profuse yellow, frothy, malodorous, pruritic discharge. A "strawberry cervix" (subepithelial redness) can sometimes be seen. The pH is between 4.5 and 6. Ceftriaxone is first line treatment for gonorrhea. The treatment for chlamydia is azithromycin or doxycycline. Candidiasis would be treated with an -azole. Staph infection could be treated by many different antibiotics other than metronidazole

Which of the following strategies help prevent bacterial meningitis?

The main bacterial causes of meningitis in children older than newborns are pneumococcus, meningococcus, and Haemophilus influenzae type B. Vaccines can effectively prevent all of these. Routinely, vaccination against meningococcus begins at age 11, though this vaccine can be given earlier for high risk children. The pneumococcal and Haemophilus vaccination series start at 2 months. Antibiotic chemoprophylaxis is recommended for household contacts of people with meningococcal and Hemophilus meningitis.

A 48 year old woman presents with frequent stools, weight loss of 8 pounds despite good intake, and heat intolerance for 6 months. Serum TSH is low, total and free T4 are high. Which medication will benefit her by blocking the synthesis of thyroid hormone?

The mechanism of action of propylthiouracil is to block synthesis of thyroid hormone. Although Graves disease is autoimmune, it is not treated using corticosteroids, making Prednisone incorrect. Propanolol decreases symptoms of hyperthyroidism by blocking beta adrenergic stimulation (thereby decreasing heart rate etc). It also decreases peripheral conversion of T4 to T3 but does not decrease overall synthesis of thyroid hormone. Levothyroixine is thyroid hormone and is used in treating HYPO, not HYPERthyroidism.

A 32-year-old healthy male was treated 4 days ago for an MRSA skin infection with sulfamethoxazole- trimethoprim (Bactrim). The infection is improving but he is increasingly weak and his sclera have turned yellow. Today his hemoglobin is 10 g/dL (13.5 to 18 g/dL) and his MCV is 85 (80 to 100 fL); the corrected reticulocyte count is elevated at 5. What is the best test for the most likely diagnosis?

The mnemonic "HIT" may assist with the differential diagnosis for hemolysis: H- Hereditary or genetic hemoglobin or enzyme issues like sickle cell, G6PD deficiency, or thalassemia I - Immune attack from antibody-antigen, best diagnosed with a Coombs test T - Trauma to red cells from fibrin threads in clots like seen in DIC. Infections in the red cells like malaria or babesia G6PD deficiency causes a hemolytic anemia when the patient is exposed to oxidant medications, like sulfa or antimalarials, or eating Fava beans. Heinz body stains will show denatured hemoglobin balls inside the red cells or "bite" cells on a peripheral blood smear. Hemoglobin electrophoresis is used for diagnosis of hereditary hemoglobin issues with a lifelong history. Erythropoietin levels are low in chronic renal disease. Ferritin levels are warranted in microcytic anemia.

A 76 yo male with presents to the clinic with 1 week of weakness and fatigue, low back pain, and increased confusion as reported by his family. He has had 2 episods of kidney stones within the past year. Which of the following electrolyte abnormalities do you suspect?

The mnemonic "stones, bones, abdominal moans, and psychic groans" describes the constellation of symptoms and signs of hypercalcemia. These may be due directly to the hypercalcemia, to increased calcium and phosphate excretion, or to skeletal changes. The presentation in a patient with hypercalcemia varies with how fast and how far the calcium level rises, as well as the sensitivity of the individual to elevated calcium levels. Mild prolonged hypercalcemia may produce mild or no symptoms, or recurring problems such as kidney stones. Sudden-onset and severe hypercalcemia may cause dramatic symptoms, usually including confusion and lethargy, possibly leading quickly to death.

A 32 year old male presents your clinic with difficulty urinating and a history of recurrent UTIs. He states that he has to push hard to urinate a few times a day. He does have a slight burning when he can get his flow going and he does not feel like his bladder empties completely. He is not sexually active currently, but does have history of Gonorrhea/Chlamydia infections x 3 in the past 6 years. What are you most concerned for?

The most common cause of urethral stricture in young males is scarring from previous STDs. This can certainly cause urinary retention with UTIs. Obstructive voiding symptoms are characterized by a decreased force of stream, incomplete emptying of the bladder, urinary terminal dribbling, and urinary intermittency. These symptoms are progressive in many patients. While urethritis and prostatitis could be possible, it is not likely unless he has active infection.

A 40-year-old female with a history of rheumatoid arthritis presents with increasing fatigue and non-vertigo dizziness when standing up. Her CBC reveals a normocytic, low reticulocyte count anemia. Which test would help determine the cause?

The most likely cause of a normocytic/ low reticulocyte count anemia in a patient with an inflammatory process (with an elevated C-Reactive protein) is increased hepcidin production by the liver. Hepcidin blocks iron transport from the liver storehouse to the bone marrow factory. Usually ferritin levels are normal, indicating no iron deficiency. The iron cannot be mobilized to make hemoglobin. The C-reactive protein level is a nonspecific indicator of body inflammation. Decreasing the inflammation will correct the anemia.

A 55 year old man presents with a red, hot, tender, swollen right knee.The pain started after he drank a six-pack of beer while watching a ball game. Past medical history is positive for hypertension for which he takes hydrochlorthiazide. CBC, serum chemistry, and uric acid are normal, you perform an arthrocentesis.Which of the following synovial findings is most likely?

The most likely diagnosis is acute gouty arthritis Risk factors are the beer ingestion and the thiazides. The knee is a common location. The normal serum uric acid level does not exclude gout. Uric acid has needle-shaped crystals in the synovial fluid; rhomboid shaped crystals indicate calcium pyrophosphate which is possible (pseudogout) but less likely.

A 30 year old woman presents with deep aching pain in her buttocks, neck, lower back, both elbows, both knees, both shoulders, and both thighs for one year. She has fatigue and states she has been sleeping poorly. Her affect appears depressed. Physical exam reveals tender points but no inflamed joints and no skin rashes or other abnormalities. Chemistry, CBC, and urinalysis are all normal. Which of the following is the most appropriate medication for this woman?

The most likely diagnosis is fibromyalgia. Drugs approved for this disorder include duloxetine, pregabalin, milnacipran. Opioids (Oxycodone) are to be avoided in chronic pain. The other drugs are for systemic lupus erythematosus.

A 65 year old woman presents with neck and shoulder pain, fatigue, and morning stiffness for two months. Physical examination shows tenderness over the right temporal region.Past medical history is otherwise unremarkable. CBC, erythrocyte sedimentation rate, and basic metabolic profile are drawn. While you await the results, which of the following is the most appropriate next step?

The most likely diagnosis is polymyalgia rheumatica. This may be associated with temporal arteritis especially given the tenderness over the right temporal artery. A complication is blindness, thus the immediate action is to give high dose steroids even before test results have returned.

Which of the following is NOT true about endometriosis?

The only way to definitely diagnose endometriosis is to visualize the endometrial implants, typically through a laparoscopic procedure.

A 57-year-old male presents with 1 month history of increasing fatigue. He is found to have a hemoglobin of 9 g/dL (13.5 to 18 g/dL) and his MCV is low at 73 (80 to 100 fL); ferritin is 7 (20 to 250 ng/mL). His vital signs and physical examination are normal. You initiate treatment with ferrous sulfate 325 mg orally three times a day. Which of the following is the next most appropriate plan?

The patient has iron deficiency anemia ( anemia, microcytosis and low ferritin level). You would begin replacement therapy and must determine why this patient is iron deficient. The correct next step is to search for the most common cause, which, in this patient's age range, would be GI blood loss. A GI work up starting with colonoscopy to rule out colon cancer is the correct next step.

A 64 year old male with a 25 pack year smoking history presents to the clinic on referral from his PCP for microscopic hematuria x 3 urine samples (6 RBCs per high powered field). He reports having regular urine flow and color. He denies dysuria, gross hematuria, nocturia, frequency or urgency. What is the best management of this patient?

The patient is high risk for bladder malignancy given his age and smoking history. 3 or more RBCs on urine microscopy is considered clinically significant and should require workup. TURBT would be too aggressive as a TURBT requires anesthesia. Rather, patient should first be scheduled for cystoscopy and CT Urogram to evaluate if there is a bladder tumor present.

In which of the following scenarios would you be most suspicious for otitis media?

The peak age incidence of otitis media is 6-12 months, and most episodes of otitis occur during or shortly after a common cold. The outer part of the ear is not ordinarily involved during an episode of otitis, and pain or tenderness of the pinna or tragus suggests an alternative diagnosis. Otitis media can cause hearing loss, but this would be very uncommon in a newborn.

A 11 year old boy develops an illness with cough, fever, red eyes, and a widespread rash. Which of the following has most increased this child's risk of contracting measles?

The single most important step in preventing measles is vaccination with two doses of MMR vaccine. Though measles has surged in some foreign countries and in some areas of the US, vaccinated individuals remain at low risk. Starting a new school, having had a transfusion, or having had chicken pox will not affect measles risk.

A 62 year-old male patient presents with 2 days of increasing right great toe pain, redness and swelling. He denies any injury to his foot. He has a history of hypertensinon and is taking a thiazide diuretic daily. On exam you see erythema, and swelling in the metatarsal-phalangeal joint of the right great toe. Your diagnosis is:

The spontaneous onset of excruciating pain, edema, and inflammation in the metatarsal-phalangeal joint of the great toe (podagra) is highly suggestive of acute crystal-induced arthritis. Podagra is the initial joint manifestation in 50% of gout cases; eventually, it is involved in 90% of cases. Podagra may also be observed in patients with pseudogout, sarcoidosis, gonococcal arthritis, psoriatic arthritis, and reactive arthritis. Rheumatoid Arthritis has persistent symmetric polyarthritis (synovitis) of hands and feet with progressive articular deterioration, extra-articular involvement and constitutional symptoms Osteoarthitis commonly effects the weight bearing joints of the knee. Septic arthritis is usually monoarticular and painful. Pseudogout is common in both genders, especially in the elderly, but is painful and usually effects large joints (i.e. knee).

Which statement about hearing is false:

The symptoms of hearing loss can be subtle, and parents may not be able to reliably detect hearing loss in their child. Vaccines prevent several causes of meningitis and mumps, which could cause hearing loss. Surgical treatment of sensorineural hearing loss with cochlear implants is now routine, and many causes of conductive loss are also treated surgically. Most states now screen newborns for nearing loss using otoacoustic emission technology, which is appropriate to use at any age.

You are seeing a child who has not received any vaccines because of his parent's personal beliefs. He is 15 months old and has stridor and respiratory distress. He is very scared, and when he tries to lie down he cannot breathe. His temperature is 38.1 degrees. Which of the following could have prevented this illness?

The vignette describes a child with epiglottitis, an illness typically caused by the bacteria Haemophilus influenzae type B. It mostly affects toddlers and young children, who present with a toxic appearance, fever, and stridor. This illness is virtually unheard of now that we routinely vaccinate against this infection.

A teenager presents with purulent tonsillitis; enlarged, non-tender adenopathy; and an enlarged spleen. Which of the following is the most likely cause of this illness?

The vignette describes a patient with clinical mononucleosis, which can be confirmed with serology or a heterophil antibody test. The most common cause of this syndrome is Epstein Barr virus (EBV). None of the other choices typically cause splenomegaly. Diphtheria is rarely seen in vaccinated individuals; adenoviral pharyngitis is often accompanied by conjunctivitis.

Which of the following is not typically seen in a Polycystic ovary syndrome or PCOS patient?

There are menstrual irregularities seen in PCOS, but amenorrhea or oligomenorrhea are typical. Obesity and insulin resistance, hirsutism, and enlarged ovaries with numerous follicles are classical signs/symptoms of PCOS. The normal 2:1 ratio of FSH/LH becomes abnormal and testosterone levels increase.

A 27 year old male sanitation worker (garbage collector) presents to the emergency department with a deep 2 centimeter length laceration over his right thenar eminence which occurred 8 hours ago at work. He has a written record of his last tetanus(Tdap) immunization 2 years ago. Based on CDC guidelines you administer:

There are two things that need to be considered in order to determine tetanus prophylaxis, the first is to determine if the wound is considered: "clean" (ie. not tetanus prone) = less than 6 hours old; linear configuration; from sharp surfaces; less than 1 cm deep, no devitalized tissue, no contaminants. or "dirty" (ie. tetanus prone) = greater than 6 hours old; stellate or avulsion configuration; greater than 1 cm deep; missile, crush, burn, frostbite; devitalized tissue; contaminants. Based on the length of time from injury (8 hours), as well as the depth and setting (garbage) this laceration would be considered "dirty" or tetanus prone. The next piece of information is when was the last tetanus vaccine given? Because the patient had Tdap within the past 5 years no prophylaxis is recommended (see CDC chart below). If it had been over 5 years then Td would be given.

Which of the following is a valid contraindication to influenza vaccination?

There are very few absolute contraindications to vaccines, but one of them is a history of a serious allergic reaction to a previous dose of the vaccination. Though some kinds of influenza vaccines are produced in eggs, the tiny amount of residual egg protein in current vaccines does not trigger a reaction even in people with a history of severe allergy. Egg allergy is not a contraindication to influenza vaccinations.

The wife of your 86 year old male patient with mild dementia calls stating that her husband is experiencing increased memory issues over the past 3 days. She states that it has been an acute change in his baseline. When asked how he is urinating she states that his urine looks fine and he is not going any more than usual. However, the caregiver states that the patient had wet his underwear last night. What are your next steps?

There is major concern for acute UTI in this patient. Often the elderly population will not complain of dysuria, malodorous urine, urinary frequency or incontinence. It often times is the spouse that notices the acute change. Prescribing empiric antibiotics is of low risk to this patient. However, allowing a UTI to fester could turn into pyelonephritis or sepsis if left long enough. Self-catheterization at home would be helpful if patient is retaining urine but that has not been proven yet and he has an active UTI.

Migraine treatment may consist of serotonin agonists (e.g. sumatriptan) known as triptans. When is this class of drugs contraindicated?

They are contraindicated in atherosclerotic heart disease or uncontrolled hypertension The safety of triptans is well established, and the risk of de novo coronary vasospasm from triptan use is exceedingly rare. However, triptans should not be taken by patients with known or suspected coronary artery disease, as they may increase risk of myocardial ischemia, infarction, or other cardiac or cerebrovascular events.

A 50 year old woman presents with polyuria and polydipsia. She states she has gained 12 pounds but only in the belly area. Physical examination shows new hypertension, facial acne, and truncal obesity. Some bruises are noted on her extremities but she denies trauma. Laboratory work shows normal serum sodium and chloride but low potassium and high glucose. Suspecting some form of Cushing syndrome/disease, you perform a low dose dexamethasone suppression test and when those results confirm your suspicions, you further evaluate with high dose dexamethasone suppression test and a plasma ACTH. ACTH is undetectable and there is no suppression of cortisol production. Which of the following is the most likely cause of this woman's disorder?

This is Cushing syndrome. Overproduction of glucocorticoids may be due to an adrenal adenoma, adrenal carcinoma, or macronodular or micronodular adrenal hyperplasia. Because of the undetectable ACTH, lung tumor is ruled out. Because of the hypokalemia and hyperglycemia, adrenal insufficiency and panhypopituitarism are ruled out.

An immunized 12 month old baby has a fever of 103 degrees for three days. A cath urine specimen showed 1+ bacteria and cefprozil was prescribed. Two days later, the fever disappeared and the urine culture returned as no growth. That night, the baby developed a macular, blanching rash. What is the most likely explanation?

This is a classic story for roseola: an infant with a few days of fever followed by a flat, blanching rash. Allergic reactions to antibiotics or ibuprofen would not typically be a macular rash. Fifth disease does not typically cause fever. Rubola is another name for measles, and would not occur typically in an immunized baby. It also would cause cough and conjunctivitis, along with a much more ill presentation.

A 55 year old man with atrial fibrillation presents with complaint of acute visual changes within the past day, "like looking through a glass of water." He denies eye trauma or headache. On exam his visual acuity is normal, and pupils are 3mm and reactive to light. Extraocular movements are intact. However on visual fields testing by direct confrontation he has a field deficit of the left visual field of each eye. How would you describe the visual field findings?

This is a homonymous hemianopsia, that is, a deficit of same half the visual field in each eye. Monocular visual loss is loss of vision of one eye. Bitemporal hemianopsia is loss of the temporal visual field in each eye, resulting in "tunnel vision." Abducens palsy is weakness of the sixth cranial nerve, the abducens. It results in an inability to abduct the eye, that is to move the eyeball laterally.

An 8 year old boy presents with a one week history of generalized edema and malaise. Vital signs are normal. His urine is foamy yellow. Which of the following is most likely to be seen on microscopic examination of his urine?

This is nephrotic syndrome due to minimal change disease (lipoid nephrosis), the most common cause in children. Oval fat bodies and Maltese cross formations are seen in the urine. Eosinophils are seen in interstitial nephritis. Red blood cell casts are found in glomerulonephritis. Rhomboid-shaped crystals are due to calcium pyrophosphate in pseudogout. White blood cell casts are seen in pyelonephritis.

A 15 year old girl presents with dark "cola" colored urine and facial edema two weeks after recovering from sore throat. Vital signs show blood pressure 156/98 though she has no prior personal nor family history of hypertension. Which of the following is most appropriate treatment for her now?

This is post-infectious glomerulonephritis.The most important management consideration is to control her blood pressure. This is NOT continuing Streptococcal infection, thus antibiotics are not indicated at this time. Steroids and immuno-suppressive agents are not indicated for this usually self-limited condition.

A 24yo female presents to the ED with a complaint of a new vaginal discharge. She has a new sexual partner. She states there is no vaginal itching or pain with the discharge, but there is a bad odor especially after sex. On clinical exam, a thin, grey discharge is seen covering the vaginal mucosa and no friability noted on the cervix. Which of the following would you expect to see under a microscope?

This is the clinical picture of bacterial vaginosis-clue cells should be visible under a microscope. Hyphae would be seen with yeast, along with more erythema of the vaginal mucosa and itching. Motile organisms would be seen with trichomonas, which would tend to cause more pain and friability of the cervix. Lactobacilli is seen in a normal, healthy vagina.

A 75 year old female presents increasing with extreme urinary frequency and urgency throughout the day for the past 3 months. She states that she is unable to go grocery shopping without having to use the bathroom while she is out. She is requesting something to help as it is interfering with her activities. What is your management?

This patient could be suffering from overactive bladder (OAB) or urinary retention with overflow incontinence. Thus, it is important to obtain a post void residual volume in clinic first to see if she is emptying. Prescribing Oxybutynin in a patient with urinary retention could make it worse. If she is emptying her bladder, oxybutynin can be very helpful for OAB patients. Caution with side effects of dry mouth and dizziness. Urodynamic testing would be helpful but not necessary to make your diagnosis. Cystoscopy not indicated.

A 41 year old woman comes into the Emergency Department with major blunt trauma to her left lateral chest wall secondary to a motor vehicle collision. On initial exam she appears to have no immediate life threatening injuries. Several minutes later her condition deteriorates. On re-examination, she is cyanotic, has a respiratory rate of 36/min, bulging neck veins, heart rate of 146/min and systolic blood pressure of 70 mmHg by palpation. There are no breath sounds on the left side and her trachea appears to have shifted to the right. The next course of action should be:

This patient has developed a tension pneumothroax (she has progressive worsening symptoms including cyanosis, tachypnea, tachycardia, hypotension, distended neck veins, no breath sounds on the left and her trachea has shifted away from the affected side) from trauma which requires immediate needle decompression and then ultimately a chest tube

A 28 year old woman who is 10 weeks pregnant presents because of tremor, nervousness, thinning hair and frequent stools. Despite good appetite, she has lost 3 lb since her initial prenatal visit. Past medical history is positive for well-controlled asthma. Physical examination shows: T 99.2 P 122 R 16 BP 112/54. She has a fine tremor and a goiter. Thyroid function tests are drawn. The labs return showing TSH 0.1 (nl 0.5-5.0), T4 48 (nl 4.6-12), T3 360 (nl 80-180). Which of the following is the best treatment for this patient?

This patient has hyperthyroidism (not very low TSH with high T3 and T4) plus her symptoms. Synthroid is thyroid hormone replacement therapy so is a treatment for hypothyroidism, not hyper. Propranolol should be avoided in obstructive lung disease (asthma) and radioactive iodine should be avoided in pregnancy. Metiamazole is usually the drug of choice but is considered teratogenic in first trimester pregnancy.

A 25 year old breastfeeding mother presents to the office complaining of a two day history of a painful, red area in her left breast. Which of the following should she NOT do?

This patient has mastitis and should continue to breastfeed or pump during treatment. Warm compresses to the affected area, antibiotics, and pain management are all beneficial in helping to resolve mastitis.

A 3 year old girl is brought to the Emergency Department after a seizure. Physical examination shows [Pictures of shortened 3rd and 4th digits on hands and feet] Labs show serum calcium 6.8 (nl 8.5-10.2) with normal serum albumin. Serum PTH is 80 (nl 10-65) Which of the following is the best long term management of this patient's condition?

This patient has pseudohypoparathyroidism. Note the low calcium despite high PTH. This indicates it is a problem with renal receptors. Pamidronate and furosemide + saline treat hypercalcemia. Parathyroidectomy is a treatment for a parathyroid adenoma, which can cause high PTH, but that is associated with hypercalcemia (and this patient has hypocalcemia). Magnesium deficiency can cause hypocalcemia but it is due to failure to secrete PTH, which is not the problem here.

A 35-year-old male presents to the ED complaining of recurrent episodes (1-2 per week, each lasting about 10 minutes) of palpitations, sweating, intense fear, shaking, tingling in his arms, and a choking sensation. The patient states these episodes occur without warning. He worries about the episodes because he is unable to prevent them, and he fears others will notice. His PE and labs are all normal. The most likely psychological diagnosis is:

This patient presents with characteristic symptoms of a panic attack. Criteria for panic disorder include the following: 1. Recurrent, unexpected panic attacks 2. Substance abuse, medication, or a general medical condition does not cause attacks. 3. Attacks followed by 1 month of one of the following: concerns about having additional attacks worry about the consequences of attacks, or a change in behavior as a result of attacks 4. Attacks are not better accounted for by another mental illness A diagnosis of obsessive-compulsive disorder entails repetitive, compulsive acts in response to recurrent, obtrusive thoughts. Diagnostic indicators for the obsessive-compulsive disorder include the following: the presence of either obsessions or compulsions, the individual is aware that the obsessions or compulsions are extreme and unwarranted, the obsessions/compulsions cause significant anguish, are time-wasting or affect the individual's life negatively. Panic disorder described as repeated panic attacks. Which are short periods of intense fear in which four or more of the following symptoms are present: fear of dying chills, sweating, chest pain, nausea, trembling, paresthesia, shortness of breath, lightheadedness, palpitations, or a feeling of impending doom. The hallmark of panic disorder is unexpected panic attacks not provoked by any particular stimulus. Patients with social phobia show a marked and persistent fear of social or performance situations, afraid that they will be humiliated or embarrassed. Affected patients will avoid these situations and can have panic attacks if forced into them. They usually realize that their fears are unreasonable. Generalized anxiety disorder is identified by irrepressible worry concerning multiple events or activities. Main features, over a period of at least six months, consist of poor concentration, lethargy, irritability, restlessness, and sleep disturbances.

A 35-year-old female admitted with a DVT and pulmonary embolus (PE) has been treated with Unfractionated Heparin UFH IV for 3 days. The CBC on day 4 reveals a normal hemoglobin, hematocrit, and WBC count, but a falling low platelet count of 70,000 (normal 150,000 to 400,000). What is the most likely reason for the drop in platelets?

This patient's platelet count dropped in half while on IV unfractionated heparin, suggesting the diagnosis of HIT- this can be confirmed with a HIT assay. The danger with HIT is that antibodies develop to platelet factor 4, inducing platelet activation with clotting and thrombosis. HUS presents with acute renal and hepatic failure with red cell hemolysis. TTP presents with evidence of multi-organ failure. ITP is usually a benign spontaneously resolving thrombocytopenia after a viral illness.

A 40 year old male presents to the clinic with ongoing pain in between his anus and scrotum. He reports that he has had the pain for years intermittently. He has been evaluated for prostatitis and all his urine cultures do not show infection. He has previously been on doxycycline for 4 weeks and didn't notice much of a difference. What is the patient's most likely diagnosis?

This scenario describes non-bacterial prostatitis. You should do microscopy of expressed prostatic secretions. If inflammatory WBCs are seen , then a short course of antibiotics (2 wk) can be given. A longer course can be given if there is symptomatic improvement, as long as the patient is counseled on increased risks of tendon rupture and the development of antibiotic-resistant bacteria. These patients would likely also benefit from additional other treatment, including alpha-blockers. If no inflammatory cells are seen, then other treatments should be tried. This succeeds approximately 50% of the time when used over a course of 4 weeks. It should be noted that antibiotics have been shown to have analgesic, antipyretic, and antiinflmmatory effects as well, and this may account for symptom improvement in some patients. If the patient does not improve with antibiotics, then another cause of symptoms must be sought and different treatment regimens must be initiated until symptoms are controlled. If the symptoms are mostly irritative (eg, dysuria with urinary urgency and frequency), then carcinoma in situ of the bladder must be excluded using urine cytology studies and cystoscopy. Helpful medicine for this patient would be NSAIDS and rest.

Which analgesic medication has aspects of opioid antagonist and antidepressant features?

Tramadol (Ultram) inhibits ascending pain pathways at the opiate receptors, altering perception of and response to pain. It also inhibits reuptake of norepinephrine and serotonin. It should not be used in patients taking antidepressants. Cases of serotonin syndrome, a potentially life-threatening condition, reported, particularly during concomitant use with serotonergic drugs; serotonergic drugs include selective serotonin reuptake inhibitors (SSRIs), serotonin and norepinephrine reuptake inhibitors (SNRIs), tricyclic antidepressants (TCAs), triptans, 5-HT3 receptor antagonists, drugs that affect the serotonergic neurotransmitter system

Which of the following is NOT true of placenta previa?

Transvaginal ultrasounds are contraindicated in placenta previa. Speculum and digital exams are contraindicated until a transabdominal ultrasound has ruled out placenta previa. All other answers are true. Tocolytics may prolong the pregnancy to allow for more growth and betamethasone can be used to accelerate lung development. If a patient has a diagnosed placenta previa at time of labor, c-section is the delivery choice.

Which of the following is NOT true regarding uterine fibroids?

True: Fibroids are estrogen dependent and are rare before puberty and shrink after menopause.

A 45-year-old male was brought to the local emergency room following a motorcycle crash. EMS reported that his leg was visibly deformed on the scene, and that his right femur could be seen protruding through his skin. At the hospital, he underwent an ORIF of his right femur and was discharged home after appropriate antibiotics, DVT prophylaxis, and physical therapy. At his 6 week post-op visit, he complains of recent fever, chills, right leg pain, and skin redness and swelling with a small area of constant drainage. Which of the following would be the least appropriate option as part of the workup? A. ESR and CRP B. Bone biopsy and culture C. Ultrasound D. CT

Ultrasound 1. ESR and CRP: while ESR and CPR are not specific for osteomyelitis, they are useful for identifying the presence of an inflammatory process 2. Bone biopsy and culture: this is the gold standard for diagnosing osteomyelitis and guiding antibiotic treatment 3. Ultrasound: ultrasound can be very helpful when examining soft tissues, but it is not very useful in evaluating bony structures 4. CT scan: this is the best modality for imaging the femur in detail, even in the presence of implants, and is critical for surgical planning

A healthy 26 year old female is in for a routine pelvic examination. She has had normal menses and no complaints of abdominal pain. You feel a right ovarian fullness and suspect an ovarian cyst. Which of the following is the preferred method of imaging to assess your patient?

Ultrasound is the preferred imaging choice to assess an ovarian cyst. If the ultrasound can't provide definitive results, then a CT scan with contrast would be the next choice.

A healthy 40 year old male patient is upset because an Xray was taken 2 months ago for suspected kidney stone and it showed nothing. The patient had to go to the ED last night with severe left flank pain and hematuria. His abdominal CT scan showed a 4mm obstructing stone at the left ureterovesical junction with moderate hydroureteronephrosis. You surmise that the stone composition is:

Uric Acid stones are not found on routine KUB Xrays. Other stones will typically show up on Xray but if your clinical suspicion is still high with negative KUB, obtain a CT scan.

A 35 year old male presents to the ED with 8 hours of right testicle swelling that increased at the end of the day. You perform trans illumination of the testicle but it does not trans illuminate. You are concerned for:

Varicocele would be the diagnosis of scrotal swelling that does not trans illuminate. This is further corroborated by the fact that it worsens at the end of the day. This can be confirmed by color doppler ultrasound of the swelling. Testicular cancer usually does not cause swelling. Epidiymal cyst also does not cause swelling.

An 18 year old college student has headache and fever. On presentation to the ER the temperature is 101.5. Exam reveals marked resistance to flexion of the neck. A lumbar puncture is performed and CSF results show normal protein and glucose, 100 red blood cell in tube 1 and tube 4, and 100 white blood cells, mostly lymphocytes. The patient has meningitis, but what type?

Viral/aseptic The hallmark of viral meningitis is lackluster CSF findings, with relatively normal protein and glucose, some elevation of white blood cells that are mostly lymphocytes. Bacterial meningitis presents with elevated protein, decreased glucose, and white blood cells mostly PMNs. Cryptococcal meningitis is proven with India ink prep of presence of cryptococcal antigen. Diagnosis of fungal meningitis is based on cultures, with elevated protein and mildly decreased glucose.

A 92 year old nursing home patient presents to the emergency department with abdominal pain and distention, vomiting and a history of chronic constipation. A plain film of the abdomen is obtained and the radiologist report states "coffee bean sign". Which of the following diagnosis is most likely?

Volvulus typically presents in older patients in their 70s, 80s, 90s, often nursing home or debilitated patients, with a history of chronic constipation; they typically present with progressively worsening abdominal pain (over hours), usually accompanied by abdominal distention; abdominal plain film radiographs often reveal "coffee bean sign"; if barium enema is utilized, then characteristic "bird's beak" or "ace of spades" finding from twisted cecum or sigmoid is seen. Hepatitis typically presents in a younger patient with RUQ pain and tenderness, organomegaly, jaundice, nausea and vomiting, no particular radiograph findings associated other than an enlarged liver. Pancreatitis often presents in patients in their 40-60s, often with a history of gallstones or excessive alcohol consumption and mid-epigastric abdominal pain that radiates to the back, with nausea and vomiting; patients often find some relief with leaning forward and/or staying very still. There are no particular radiograph findings associated other than maybe some calcifications in the pancreas or a gallstone ileus. Nephrolithiasis often presents in males with sudden, severe, abdominal pain that radiates to the ipsilateral groin, often are writhing in pain in an attempt to find relief; no particular radiograph findings associated other than maybe some calcifications in the ureter or kidney. Small bowel obstruction often presents in those with previous abdominal surgery secondary to adhesions; abdominal exam often reveals distention, pain on palpation (guarding, possibly rebound tenderness), and auscultation reveals hyperactive bowel sounds with high pitched tinkling, metallic sounds with air rushes. With complete obstruction or bowel fatigue, patients may have periods of absent bowel sounds. Radiographic findings on plain film may include bowel distention, "stack of coins" and multiple air fluid levels.

A 16-year-old female has a 6 month history of increased menstrual bleeding, measured by both the number of days and quantity of pads. Her Gyn exam is normal. Her CBC reveals a low reticulocyte count microcytic anemia with a normal WBC and platelet count. Coagulation studies were done. The clotting time/PFA is prolonged and her PT is normal. The aPTT is prolonged and corrects with a mixing study. What is the most likely cause of the increased bleeding?

Von Willebrand factor (vWF) has two functions: to stick platelets to broken blood vessel walls (super glue) and stabilize factor VIII. If vWF is low, as in Von Willebrand disease, you would see a prolonged platelet function analysis (PFA) and a prolonged aPTT (decreased Factor VIII because not enough stabilizer). In true genetic Hemophillia A or B, the platelets work normally with a normal PFA, but just the aPTT would be prolonged. The aPTT corrected with normal serum so there would not be any factor antibodies present.

A 34 year old 6 month pregnant female presents to ED after high speed motor vehicle accident where she was wearing her seatbelt. She has no vaginal bleeding and good fetal heart tones. She is complaining of lower abdominal pain. What injury are you most concerned for?

While spontaneous abortion is certainly concerning, she is not having any vaginal bleeding at this time. Rather the majority of bladder ruptures and injuries in trauma occur because of motor vehicle accidents and the position of the seat belt.

The definitive treatment for adenomyosis is:

With adenomyosis, hysterectomy is indicated because the inner muscle of the uterus is affected. Endometrial ablation, D&C, hysteroscopy, and OCP's tend to be more beneficial with endometrial lining issues. Patients with adenomyosis will commonly report symptoms similar to those reported by patients with endometriosis. Common complaints include menorrhagia, dysmenorrhea, metrorrhagia, chronic pelvic pain and dyspareunia. Adenomyosis is defined as the presence of ectopic nests of endometrial glands and stroma throughout the myometrium, surrounded by reactive hypertrophic smooth muscle cells.

A 52 year old chef sustains scald burns from a large pot of boiling water that overturns onto his entire anterior chest and abdomen and anterior thighs bilaterally. His genital region and lower legs are spared. Using the rule of 9s, which of the following is the most accurate estimation of body surface area involved?

anterior chest/abdomen= 18% bilateral thighs= 4.5% each since each entire lower ext is 18% circumferentially which is 9% anterior (thigh/lower leg) and 9% posterior (thigh and lower leg) add up to 27%. The "rule of nines" is less accurate in children because their body proportions are different from those of adults.For areas of irregular or nonconfluent burns, the palmar surface of the patient's hand can be used. For a wide age range, the area of the palm without the fingers represents 0.5% of the body surface.

A seventeen year old soccer player bumped her head with another player, falling to the ground in a dazed condition, without loss of consciousness. On the sideline, she was confused about the details of the game and she sat out the rest of the game. On evaluation the next day she was diagnosed with a concussion. What is the most important component of treatment plan?

physical and mental rest The most important aspect of concussion treatment is physical and mental rest to reduce the metabolic demands on the injured brain. Good sleep, hydration and nourishment are also important, along with avoiding neurotoxins like alcohol. Psychosocial support and normalizing the experience are important in virtually every aspect of medicine, but are especially vital in the care of a child with Tourette's syndrome, for example. Brain imaging is necessary in the concussed patient who has other general neurological findings like lethargy, agitation, confusion, or focal findings like slurred speech, hemi-weakness, staggering gait, double vision, etc., or in the patient who is not getting better after a few days.


Ensembles d'études connexes

ATI Dosage Calculations Critical Care Medications

View Set

Lesson 17: Real Estate Careers and The real estate license law

View Set

Relative Dating of Rock Layers by Principles

View Set

Parkinson's Disease Practice Questions

View Set

Physics Chapter 22 mastering physics

View Set

Business Finance Final Exam (Chapter 8)

View Set

Chapter 18: Inferences about Means

View Set